Jump to content

Wikipedia:Reference desk/Humanities: Difference between revisions

From Wikipedia, the free encyclopedia
Content deleted Content added
Line 473: Line 473:
:::See [[Dan Bullock]] ''"...the youngest American serviceman killed in action during the Vietnam War."'' He joined the Marine Corps at 14 and was killed at An Hoa Combat Base aged 15. [[User:Alansplodge|Alansplodge]] ([[User talk:Alansplodge|talk]]) 19:34, 28 October 2012 (UTC)
:::See [[Dan Bullock]] ''"...the youngest American serviceman killed in action during the Vietnam War."'' He joined the Marine Corps at 14 and was killed at An Hoa Combat Base aged 15. [[User:Alansplodge|Alansplodge]] ([[User talk:Alansplodge|talk]]) 19:34, 28 October 2012 (UTC)
::::See also [http://turnersolutions.jigsy.com/entries/general/my-father-s-story-by-crissandra-l-turner My Father's Story by Crissandra L. Turner], which says of Edward Turner Jr. ''"In 1953 he sneaked off and joined the army Airborne using a cousin’s name at the age of 14, making him the youngest Korean War Veteran from the State of Missouri."'' [[User:Alansplodge|Alansplodge]] ([[User talk:Alansplodge|talk]]) 20:12, 28 October 2012 (UTC)
::::See also [http://turnersolutions.jigsy.com/entries/general/my-father-s-story-by-crissandra-l-turner My Father's Story by Crissandra L. Turner], which says of Edward Turner Jr. ''"In 1953 he sneaked off and joined the army Airborne using a cousin’s name at the age of 14, making him the youngest Korean War Veteran from the State of Missouri."'' [[User:Alansplodge|Alansplodge]] ([[User talk:Alansplodge|talk]]) 20:12, 28 October 2012 (UTC)

:The movie ''[[Too Young the Hero]]'' is the true story of a 12-year-old who joined the Navy during World War II. [[Special:Contributions/69.62.243.48|69.62.243.48]] ([[User talk:69.62.243.48|talk]]) 00:26, 29 October 2012 (UTC)


== how much cropland is lost to circle farming? ==
== how much cropland is lost to circle farming? ==

Revision as of 00:26, 29 October 2012

Welcome to the humanities section
of the Wikipedia reference desk.
Select a section:
Want a faster answer?

Main page: Help searching Wikipedia

   

How can I get my question answered?

  • Select the section of the desk that best fits the general topic of your question (see the navigation column to the right).
  • Post your question to only one section, providing a short header that gives the topic of your question.
  • Type '~~~~' (that is, four tilde characters) at the end – this signs and dates your contribution so we know who wrote what and when.
  • Don't post personal contact information – it will be removed. Any answers will be provided here.
  • Please be as specific as possible, and include all relevant context – the usefulness of answers may depend on the context.
  • Note:
    • We don't answer (and may remove) questions that require medical diagnosis or legal advice.
    • We don't answer requests for opinions, predictions or debate.
    • We don't do your homework for you, though we'll help you past the stuck point.
    • We don't conduct original research or provide a free source of ideas, but we'll help you find information you need.



How do I answer a question?

Main page: Wikipedia:Reference desk/Guidelines

  • The best answers address the question directly, and back up facts with wikilinks and links to sources. Do not edit others' comments and do not give any medical or legal advice.
See also:



October 22

women priests

In Judaism, Reform Jews and Conservative Jews and Reconstructionist Jews have rabbis who are women. Is Judaism the only religion in the world that have female priests in specific sects? No debate, just simple answer like no or yes. Thanks.--70.31.22.91 (talk) 01:16, 22 October 2012 (UTC)Don Mustafa[reply]

No. See Ordination of women. AndyTheGrump (talk) 01:18, 22 October 2012 (UTC)[reply]
Rabbis are not priests and certainly not priestesses. See http://www.jewfaq.org/rabbi.htm μηδείς (talk) 02:02, 22 October 2012 (UTC)[reply]
To summarise the Christian situation briefly - there's a complete spectrum. The Anglican communion is divided on the subject. The Presiding Bishop of the Anglican Church in the USA is a woman (and also a priest, necessarily). The Church of England is just getting set to create its first women bishops, but has had women priests for some time. The Lutheran Church of Sweden has women bishops too - the Bishop of Stockholm, for example.
The Roman Catholic church and the various Orthodox churches do not have women priests.
Not all Christian churches have priests at all. AlexTiefling (talk) 09:06, 22 October 2012 (UTC)[reply]
And, of course, women played and play important rules in all kinds on non-Christian religions. See e.g. Vestal Virgin and Pythia for classical examples. --Stephan Schulz (talk) 10:30, 22 October 2012 (UTC)[reply]
I found Religious sexism: when faith groups started (and two stopped) ordaining women. Alansplodge (talk) 10:34, 22 October 2012 (UTC)[reply]
Some religions, unlike the Abrahamic religions, explicitly promote the elimination of prejudice. See Baha'i Faith and the unity of humanity. Baha'i has no priests (and considering that rabbis aren't priests, the OP's requirements are probably not strict), but its elected councils are open to both men and women. --140.180.242.9 (talk) 18:34, 22 October 2012 (UTC)[reply]
Not true, 140. The highest elected council, the Universal House of Justice, is definitely not open to women of any age, experience or wisdom. A 21-year-old male is, however, eligible. Bielle (talk) 18:46, 22 October 2012 (UTC)[reply]
In any case - in what sense is Baha'i not 'Abrahamic'? AlexTiefling (talk) 19:28, 22 October 2012 (UTC)[reply]
@Bielle: You're most definitely right. I was somehow under the impression that it wasn't an elected council.
@Alex: It's not Abrahamic in the sense that it doesn't claim to have originated from Abraham and considers numerous religions to be "true", many of which have no relation to Abraham and explicitly claim another founder. In that sense, they're similar to delusional liberals who believe all religions to be true while ignoring the fact that all of them are ludicrous, self-contradictory, and mutually contradictory. --140.180.242.9 (talk) 04:45, 23 October 2012 (UTC)[reply]
Nevertheless, it developed from a form of Islam (Shaykhism/Babism), which is usually enough to count as "Abrahamic", unless there has been radical discarding of historical influences... You can look at The Abolition of Man by C.S. Lewis (a determined Christian who definitely did not think that all religions are "true") for a systematic defense of the idea that almost all religions contain valid moral foundations... AnonMoos (talk) 07:46, 23 October 2012 (UTC)[reply]

@all. While it is true that the House of Justice is limited in it's membership it is insufficient to summarize the Baha'i principles on the equality of the sexes with just that fact. Individually members of the House are not seen as the highest individual office - that would be Continental Counsellors and yes there have been women. Before that institution there were Hands of the Cause and again there were women - indeed the most prominent in the 20th century were women (see Martha Root, and Rúhíyyih Khanum .) Additionally the principles-in-action represented by the clear prominence of women at the national level outstrips all other international organizations and almost every (I don't know of one, but who knows) national organization in freely voting for a higher percentage of women to office holders. As for priests - yes there are none and that goes with being intermediaries in religious rites to be "right" with God - as there are none of the former there are none of the latter. And in particular to being Abrahamic - it is true the religion acknowledges non-Abrahamic religions this does not invalidate it as Abrahamic. Obviously it acknowledges Abraham and claims to have descend ended from two of his wives, and there are reviews that make many parallels in teachings between the religions as is detailed in the Abrahamic religions article. As for not taking seriously that there are problems of interpretation - well there are two parts to that -taking it seriously, and wrestling with those differences. On the first part let me offer, in another connection, that my father in law was a conscioustous objector to WWII and that during his time in the forest of Oregon in a work camp he became a Baha'i and then had to go through boot camp and served as a non-combatent in war as a medic. I can't say all Baha'is take all things seriously, but clear some do and it matters. I've not seen a systematic study of how Baha'is wrestle with the contradictory approaches taken within and among all the religions but a common thought and one you can find in a religion-by-religion analysis is that if you look at the original scriptures you can find bridges of understanding that resolve differences others later made into particular understandings and lost the ability to think again what they meant. This is why - in the Abrahamic lineage - you have religion after religion, rather than a succession of one religion (similarly in the system of eastern faiths with Hinduism to Buddhism which could have been a succession of one religion.) But that would not be possible today with the administrations of each religion. Smkolins (talk) 12:03, 26 October 2012 (UTC)[reply]

Who is the person Obama is bowing to?

Who's Obama bowing to in this picture? Thanks. μηδείς (talk) 01:59, 22 October 2012 (UTC)[reply]

Viktor Yushchenko ---Sluzzelin talk 02:07, 22 October 2012 (UTC)[reply]
Wow, he looks great since the poisoning. μηδείς (talk) 03:05, 22 October 2012 (UTC)[reply]
Yuschenko's disfigurement was primarily chloracne and bloating, both of which fade over time. You can see the fading over the years — 2006 (bloating and chloracne) 2007 (just chloracne) 2009/2009 (much faded chloracne). (Compare before/after in 2004). I suspect some of his scarring may be permanent but it's gotten a lot better. --Mr.98 (talk) 16:54, 22 October 2012 (UTC)[reply]
It's not unambigiously clear that Obama is in fact bowing. It's quite possible that he is meley looking at and commenting on Yushchenko's shoes. Roger (talk) 15:32, 22 October 2012 (UTC)[reply]
Maybe he's both bowing and shaking hands at the same time, as he did much to the amusement of the incumbent Mikado?--Wehwalt (talk) 15:41, 22 October 2012 (UTC)[reply]
Yes... bowing and shaking hands at the same time. Obama is known for doing this when greeting other heads of state (especially when he is the visitor to their country). Some criticize him for this practice (seeing the bow as a sign of submission, which should not be done between equals). Others don't have a problem with it (seeing the bow as a sign of respect, perfectly acceptable between equals). Blueboar (talk) 15:45, 22 October 2012 (UTC)[reply]
Obviously he's about to pull the old "your shoelace is untied" gag. Clarityfiend (talk) 21:07, 22 October 2012 (UTC)[reply]

Amazons

According to our article on the Amazons:

"According to ancient sources, (Plutarch Theseus,[74] Pausanias), Amazon tombs could be found frequently throughout what was once known as the ancient Greek world. Some are found in Megara, Athens, Chaeronea, Chalcis, Thessaly at Skotousa, in Cynoscephalae and statues of Amazons are all over Greece. At both Chalcis and Athens Plutarch tells us that there was an Amazoneum or shrine of Amazons that implied the presence of both tombs and cult. On the day before the Thesea at Athens there were annual sacrifices to the Amazons."

As far as I know, the Classical Greeks thought the Amazons were humans, not goddesses. Not only that, they were enemies of Greece and never had a single friendly encounter. So why were the Amazons worshipped? --140.180.242.9 (talk) 06:58, 22 October 2012 (UTC)[reply]

Remember that to the ancient Greeks, the Gods were just normal humans who happened to be immortal. And there were those in-between, like Hercules. So, they could think of the Amazons, at various times, as legendary humans or as goddesses. StuRat (talk) 07:08, 22 October 2012 (UTC)[reply]
140.180.242.9 -- In ancient Greek culture, local ancestors and culture heroes / "demigods" were honored, and sometimes sacrificed to, without being confused with gods as such... AnonMoos (talk) 12:51, 22 October 2012 (UTC)[reply]
But the amazons were neither the ancestors of the Greeks nor demigods. The only full mortals I know of who were sacrificed to were legendary founders (and Greeks) like Perseus. --140.180.242.9 (talk) 18:23, 22 October 2012 (UTC)[reply]
They weren't particularly divinities, but they were semi-mythical figures with striking personal characteristics, and if some of them developed local associations, they might be considered "heroes" in a sense... AnonMoos (talk) 00:50, 23 October 2012 (UTC)[reply]

Whitecollar : 2 cultural questions please

Hello L.O. (Learned Ones) ! The TV serie Whitecollar is gaining audience in France, thanks to its aesthetic qualities and winding intrigues, & the IIIrd season, 14° episod (Pulling Strings) has been welcomed : bound to be, it features Matt. Bomer as a beloved character of ours : Arsène Lupin (or at least as his great-grand-son) , and a good value for us here, a Stradivarius violin. Could you give me some cultural references tips about :

1/ the doll retrieved after 30 years : great round sullen eyes, concave profile with sunken little nose, stiff bleached hair, limp body, makes you think of an abused off-spring from alcoolic parents... I don’t remember having seen such a freak in little girls’s hands during the ’80. Was this doll à la mode in the US then , did it have a specific name ?

2/ I didn’t see clearly what New York concert hall Neal and Sara were attending : a wide amphitheatrum with big low armchairs, with mustard-colored (I think) walls and furniture. Can you tell me what theater it was ? Thank you beforehand for your answers, T.y Arapaima (talk) 08:50, 22 October 2012 (UTC)[reply]

1) Cabbage Patch Kids: [1] ? They were quite the fad in the 1980s. StuRat (talk) 09:11, 22 October 2012 (UTC)[reply]
Looking at screenshots (ETA: here) it doesn't seem to be a cabbage patch kid. --NellieBlyMobile (talk) 16:02, 22 October 2012 (UTC)[reply]
Blimey! It bears a slight resemblance to a Troll doll, but I can only find pictures of smiling ones.[2] Alansplodge (talk) 16:21, 22 October 2012 (UTC)[reply]
It rings no bells with me. This wouldn't be the first fictional fad. —Tamfang (talk) 18:30, 22 October 2012 (UTC)[reply]
According to [3] it's a Little Miss No Name, a Hasbro line from 1965. (What, do we not have an article?) Marnanel (talk) 20:36, 22 October 2012 (UTC)[reply]
Could this be the concert hall? That's the Metropolitan Opera House at the Lincoln Center for the Performing Arts. (ETA Or the Avery Fisher Hall?) --NellieBlyMobile (talk) 16:02, 22 October 2012 (UTC)[reply]
Thanks a lot to you all for your answers ! About the "Little Miss No Name", I fairly understand some children threw it away, as told in the Whitecollar episode...Arapaima (talk) 17:42, 7 November 2012 (UTC)[reply]

Automatic Pistols

All the (semi)automatic pistols I have seen require manual loading of first cartridge, i.e. after inserting a loaded magazine you pull and release the bolt slide so that first cartridge gets into breech, then you fire and need not pull bolt next time, hence "automatic". My question is that are there any models that do not require the loading of first bullet ? If so then how does it manage to get first cartridge from magazine into breech ? 124.253.95.164 (talk) 09:56, 22 October 2012 (UTC)[reply]

Answer to first question - No. Answer to second question - Moot (see answer to first question). All automatic weapons invented so far require a deliberate action by the operator to move the first round from the feed mechanism into the chamber. This applies equally to hanguns, rifles, machineguns all the way up to large automatic cannon. Roger (talk) 13:15, 22 October 2012 (UTC)[reply]
The question reminds me of a funny blunder in The Killing (Kubrick's heist movie, 1956): Elisha Cook picks up an empty pistol, works the slide and then inserts a magazine, leaving the chamber empty.
A quibble worth noting imho: With most pistols, when you fire the last round of a magazine, the slide stays open. You then replace the magazine and release a catch, and the slide closes, loading the first round with less work on your part. —Tamfang (talk) 18:28, 22 October 2012 (UTC)[reply]
I know that, of course. I know my bit about guns, especially autos, to know that " All automatic weapons invented so far require a deliberate action by the operator to move the first round from the feed mechanism into the chamber. This applies equally to hanguns, rifles, machineguns all the way up to large automatic cannon." , as told above. That is the reason I am asking this question ! A vendor told me about a .25 that works (he claims) thus ! and we are not talking about the bolt held back by previously exhausted magazine as Tamfang has told... — Preceding unsigned comment added by 124.253.60.148 (talk) 05:51, 24 October 2012 (UTC)[reply]
Not always a good thing, because it tells everyone that you've fired your last shot. Alansplodge (talk) 12:03, 23 October 2012 (UTC)[reply]

Last living veteran of the Second Anglo Afghan War?

Last living veteran of the Second Anglo-Afghan War? Thank you. Iowafromiowa (talk) 10:19, 22 October 2012 (UTC)[reply]

The best I can do is; The Second Anglo-Afghan War 1878-1880 which links to Yahoo Groups: The Second Anglo-Afghan War and says that "Some topics already mentioned include... longest living Afghan war veterans." You have to join the group to read the messages. Alansplodge (talk) 13:13, 22 October 2012 (UTC)[reply]
I should have known; WHAAOE. See List_of_last_survivors_of_historical_events#1850–1900; "Hugh Theodore Pinhey - February 6, 1953 (aged 96) - Last veteran of the Second Anglo-Afghan War." Alansplodge (talk) 13:17, 22 October 2012 (UTC)[reply]
That's an amazing list. It even names the "last individual who claimed to have spoken to Virgin Mary." No more claimants then?--Shantavira|feed me 14:57, 22 October 2012 (UTC)[reply]
That's surprising to me. Do Catholics no longer ask her to do something or other now and at the hour of their deaths amen?--Wehwalt (talk) 15:43, 22 October 2012 (UTC)[reply]
Not just Catholics... although, apparently she does not reply as often as she used to do. Perhaps the entry should be changed to "last individual to claim that the Virgin Mary spoke to him/her" Blueboar (talk) 15:54, 22 October 2012 (UTC)[reply]
That's not right either, since I'm sure there are many mentally ill people who have imagined that she's spoken to them. Perhaps "last individual believed by the Roman Catholic Church to have spoken with the Virgin Mary"? This also avoids the implication that Wikipedia is judging the merits of any claims. --NellieBlyMobile (talk) 17:13, 22 October 2012 (UTC)[reply]
Even more amazing is that whenever Joe and Rose Kennedy had a child, it's regarded as a historical event. -- Jack of Oz [Talk] 18:51, 22 October 2012 (UTC)[reply]
That 1917 Fatima claim is wrong, anyway. According to Marian apparition, the Catholic Church has accepted the claims of later apparitions and conversations with the Virgin Mary: in Beauraing 1932/33, Banneux 1933, and Akita 1973. -- Jack of Oz [Talk] 18:58, 22 October 2012 (UTC)[reply]
Oh, I see what's going on. It's saying that Lúcia Santos was the last survivor of a conversation with Mary (1917), because she lived till as late as 2005. I'm still not sure that's a correct claim. Sister Agnes Sasagawa, who conversed with Mary at Akita in 1973, has not to my knowledge died, so she has outlived Lúcia Santos. -- Jack of Oz [Talk] 20:17, 22 October 2012 (UTC)[reply]
I guess they must meant that she was the last of the three children to see her at (edit) Fatima. I've edited the list to be clearer. --NellieBly (talk) 23:14, 22 October 2012 (UTC)[reply]
Thanks, Nellie. -- Jack of Oz [Talk] 23:33, 22 October 2012 (UTC)[reply]

More about our man Hugh; a long letter that he wrote to his mother describing the Siege of Kandahar can be seen here. "Hugh Pinhey was an Assistant Superintendent in the telegraphy department, but for the duration of the siege was given the rank of lieutenant and assigned to the 4th Bombay Rifle Corps". Alansplodge (talk) 16:45, 22 October 2012 (UTC)[reply]

A serious flaw in any "Last living veteran of the <"civilized country"> versus <"savage tribe"> wars" list, is that reliable information exists for the veterans of only one side. In this particular case - who was the last Afghan veteran? Roger (talk) 14:23, 24 October 2012 (UTC)[reply]
Agreed, perhaps "last British veteran" would be more accurate. Alansplodge (talk) 22:02, 24 October 2012 (UTC)[reply]

Oath of Office of the President of the United States

Since this is the Oath of Office of the President of the United States [the district], an employee of the congress under the constitution of the US of 1789 and the chief administrator for the united States of America. What information is available regarding the "Oath of office for the President of the United States of America" under the Articles of Confederation 1777(?) as found in the present/current United States Code????

Thank you, robert: carr — Preceding unsigned comment added by 208.58.114.72 (talk) 17:21, 22 October 2012 (UTC)[reply]

Under the Articles of Confederation there was no such office as President of the United States. The nearest thing was the President of the Continental Congress, but that was a far less powerful office, having a one year term and very limited authority. The Articles did not specify an oath for that office. Looie496 (talk) 18:28, 22 October 2012 (UTC)[reply]
Indeed. The President of the Congress was basically the Speaker; he was in charge of maintaining order at the meetings of Congress and almost nothing else. There was no single Chief Executive of the U.S. under the Articles of Confederation (also no unified currency, barely any military, no federal judiciary, etc. AFAIK, all they did was send ambassadors abroad to secure loans and trade agreements and not much else). --Jayron32 22:41, 22 October 2012 (UTC)[reply]
Being a member of the Continental Congress was a thankless and largely powerless office, and being the President of the Continental Congress also had its drawbacks. It is very easy to envision someone being elected, but not wanting to take on the office (Poor health, etc). Thus there was a need for some action to be taken at some point in time whereby the electee actually accepted and assumed the office. Journals of the Continental Congress were preserved. So what words were uttered, what actions were taken, and how was the action logged in the journal of the Continental Congress by the Secretary thereof? Edison (talk) 02:53, 23 October 2012 (UTC)[reply]
A volume of the Journal says that the secretary and clerks had to take "an oath of fidelity to the United States and an oath of fidelity for the execution of their respective trusts." It is likely that the president had to take similar oaths. Edison (talk) 03:00, 23 October 2012 (UTC)[reply]
Possibly not. As a delegate from his state, the president was already an elected official and a member of Congress, so a new oath may have been superfluous. The secretaries and clerks were new hires, however. —Kevin Myers 00:46, 24 October 2012 (UTC)[reply]

Kippah type fabric Reconstructionist

In the Kippah article, you mentioned the type of kippahs for different sects of jewish and the fabric. What about Reconstructionists? Which fabric do they use for kippahs? — Preceding unsigned comment added by 70.29.34.207 (talk) 17:29, 22 October 2012 (UTC)[reply]

You'll get a better chance of a good answer by posting this question at Talk:Kippah. You might want to crosspost a link to that question at Talk:Reconstructionist Judaism, but please don't cross post by putting up the question twice. --Dweller (talk) 09:04, 23 October 2012 (UTC)[reply]

sects protestantism

What are the main sects of Protestantism practiced in the world today in USA, Sweden, Norway, Iceland, Germany, Netherlands, Denmark, South Africa, Anglophone Caribbean, UK, Australia and New Zealand? — Preceding unsigned comment added by 70.29.34.207 (talk) 18:03, 22 October 2012 (UTC)[reply]

By sect, do you mean branch/movement or are you referring to denominations? Norway, Sweden, Finland, Denmark and Iceland each have a very large state/national church, which is by far the largest denomination in their resprective countries. All of these are Lutheran, but few of the members actually attend church services. Germany likewise has mainly Lutherans, although these are divided into many individual (though often cooperating) churches. The Protestant Church in the Netherlands is the largest protestant denomination in the Netherlands. This church and most other Dutch protestant churches originated from the Dutch Reformed Church, and are mostly rooted in reformed (calvinist) theology. I'll let others address the other countries. - Lindert (talk) 18:17, 22 October 2012 (UTC)[reply]
Your desired data for the US are available at Christianity in the United States, with a comprehensive list of like articles at Christianity by country. — Lomn 18:49, 22 October 2012 (UTC)[reply]

Maximilian Kolbe

Hello,

I am curious about the article Maximilian Kolbe. What happened that it was viewed 56000 times in one day and was vandalized almost consequently? Just curious. Regards.--Tomcat (7) 18:21, 22 October 2012 (UTC)[reply]

That was October 10. The article appeared on the main page in the "On this day" area, marking the 30th anniversary of his elevation to sainthood. Looie496 (talk) 18:33, 22 October 2012 (UTC)[reply]

how are saints made?

Yesterday on the radio I heard several times that someone was "made" a saint. Am I right in thinking that canonization (at least in Rome) doesn't make a saint but rather recognizes that someone is a saint? —Tamfang (talk) 18:36, 22 October 2012 (UTC)[reply]

You are correct, but the point is a technical one, and it's almost certain the reporter got it wrong. AlexTiefling (talk) 18:39, 22 October 2012 (UTC)[reply]
Saints are not made, they are born. Now seriously, just check canonization for more details, you just get recognized to be a saint. 80.39.52.222 (talk) 20:58, 22 October 2012 (UTC)[reply]
If you can "install" a pope without the aid of a rivet gun, it only stands to reason that you can "make" a saint without a factory. :-) StuRat (talk) 22:37, 22 October 2012 (UTC) [reply]
I just figured the radio reporter had confused saints with the British system of titles of nobility, or possibly the mafia. ←Baseball Bugs What's up, Doc? carrots22:41, 22 October 2012 (UTC)[reply]
Speak softly, Bugs. They won't want to hear that. --NellieBly (talk) 23:02, 22 October 2012 (UTC)[reply]
"They" being Saints, Peers of the Realm or mafiosi? Surely a conjunction rarely found in one sentence. Alansplodge (talk) 00:09, 23 October 2012 (UTC)[reply]
Maybe "created" is the more commonly used term in titles of nobility. "He was created the Earl of Oil", or whatever. In the underworld a "made guy" has committed his first murder for the family business. It's curious to think of a saint as being a "made guy". ←Baseball Bugs What's up, Doc? carrots05:12, 23 October 2012 (UTC)[reply]
I'm suddenly reminded of The Bishop. Dismas|(talk) 04:57, 23 October 2012 (UTC)[reply]

Christian traditions...

Where and how can I find information about specific methods or beliefs of different Christian intepretative traditions? Often, people use the general term "in some traditions" without nailing on which tradition they may be referring to, thereby drawing a blank. Even the Wikipedia article Sacred Tradition talks about how this tradition is passed down orally, in the lives of Christians, thereby giving the impression that the only way to understand Christianity fully is to meet a Christian in person and assume that Christian knows/understand scripture as well as tradition passed down by his/her ancestors or learned from the church. Furthermore, the Christian would presumably use the conversation as an opportunity to proselytize the non-Christian, making the Christian life and faith attractive in every way to the point of bringing in more converts. How do Christians know what traditions to reject and what traditions to accept and what traditions are ought to be modified, or do they just accept whatever is part of their church without question? This may hint at the pejorative connotation of the word indoctrination. 140.254.227.51 (talk) 18:58, 22 October 2012 (UTC)[reply]

Well, would you take it from me that I'm a practising member of the Church of England, that I have no interest in converting you, and that I know a reasonable amount about my own tradition and others?
It's important to understand the distinction between a tradition and a denomination. For example, the high church tradition is found in Anglicanism and in Lutheranism. Those denominations are separate (although some of their constituent churches are linked by the Porvoo Agreement), and each contains many congregations (a sizeable majority) who do not represent that tradition - but also many who do.
And it's not just about scripture and the interpretation of scripture. Indeed, when the Roman Catholic church refers to its own three teaching traditions, only one of those is scripture. The others are reason and tradition itself. The idea that Christian life and worship is all about the Bible is itself the product of one particular tradition - which is often called fundamentalist, but might more properly be called evangelical.
So if you have specific questions, please ask. AlexTiefling (talk) 19:18, 22 October 2012 (UTC)[reply]
The fundamentalist tradition would fit under the evangelical tradition, which would be a subset of Protestantism, as many evangelicals use the Protestant Bible (66 books, no Apocrypha). With so many Christian traditions around, it is easy for one particular denomination to think that that denomination is the one true church or one true faith and thus is superior than the other churches or denominations. Some Christians treat Christianity as a big-marketing strategy to help emotionally/socially isolated people or social outcasts or impoverished people in order to convert these people to Christianity. Also, given that some churches explicitly describe on their church websites that "salvation is free", they are hinting that anybody can become a Christian and receive God's "free gift", even though they may be forgetting that they are expecting that the Christians ought to behave what the church wants them to behave. 140.254.227.51 (talk) 20:00, 22 October 2012 (UTC)[reply]
(EC) :"Tradition" is a word that is often used nowadays to mean Christian denomination (our article uses the word twice in the first paragraph). It's a move away from "this is our faith and other Christians are wrong" towards "this is our faith, others have different traditions". So I suspect "in some traditions" means "in some denominations". Use of the word "church" in this meaning can be confusing, as it can refer to a) all Christians, regarded as "Christ's body on earth" Saint Paul's First Letter to the Corinthians, Chapter 12: verses 12-27 b) a particular denomination c) a subset within that denomination as the Church of England and the Church in Wales within Anglicanism d) a particular local congregation or e) an actual building. "How do Christians know what traditions to reject and what traditions to accept"? The most obvious answer is that they usually go to church on Sunday (see meaning "e") where those traditions are enacted, and explained during the sermon. Alansplodge (talk) 19:35, 22 October 2012 (UTC)[reply]
So, the only way to see the traditions enacted or come to life is to actually enter a church, or if you are a non-Christian, pretend to be one and enter a church, so that you can witness the sermon, the style, the manner, et cetera, and compare that experience to all Christian churches you may have visited in your local area - which, again, can be quite subjective - in order to track down what a particular church believes in. I suppose one needs to keep one's eye open when walking inside a church to see what others are doing and that person does the same. In Western churches, everyone does the same thing at the same time, which to Eastern churches would seem very regimented. In Eastern churches, everyone does different things at the same time and stand up to worship, which to Western churches would seem very disorganized. The non-Christian visitor would somehow have to find a way to keep that in mind and make an attempt to blend in with the crowd. Sort of like Waldo. 140.254.227.51 (talk) 19:50, 22 October 2012 (UTC)[reply]
You will not have to pretend to be a Christian at all. Usually there are no restrictions for attending a service/sermon. If anyone asks, just tell them you are interested to learn about their faith. You could also ask if you can speak to the pastor or to an elder sometime after the service to learn more, or you can find the church's website, which will usually contain a statement of faith and more. - Lindert (talk) 20:13, 22 October 2012 (UTC)[reply]
(after ec) Given that you asked this question, you've also done an awful lot to answer it too - mostly by delivering a sermon of your own. The short answer to your original question 'where and how can I find information' is (obviously) right here, because Wikipedia is an encyclopedia. But I made a genuine offer to answer specific questions about my own tradition and those in which I've studied. If you've got questions, ask away. I'll try to back my answers up with links and so on where possible. AlexTiefling (talk) 20:14, 22 October 2012 (UTC)[reply]
OK. There was this organization on campus that offered Bible study sessions every Wednesday. So, I attended a couple of times, and during my time there, it seemed that the group derived its interpretations on annotations made by "Living Stream Ministry". I looked up the ministry on Wikipedia, and noticed that it was an evangelical Christian ministry. Nevertheless, the group displayed on its website that it was "interdenominational", accommodating all denominations, not just "evangelical". One time, the group had these tiny pieces of paper that advertised "The Recovery Version" of the Bible, specifically the New Testament. How trustworthy or representative of Christianity are evangelicals? Do they just represent a non-denominational Protestant tradition? 140.254.227.51 (talk) 20:48, 22 October 2012 (UTC)[reply]
Yeah, that would be a non-denominational evangelical Protestant group. For the maximum amount of amusement, you should tell them you're Roman Catholic. Adam Bishop (talk) 21:01, 22 October 2012 (UTC)[reply]
Just to clarify, most evangelicals are not non-denominational. The answer on whether they represent Christianity well will obviously differ on whom you ask. - Lindert (talk) 21:34, 22 October 2012 (UTC)[reply]
(e/c) Going to an organization that offers Bible study lessons might not really be the best idea. There's a good chance you'll be surrounding yourself with zealots. You'll immediately stand out as a new face/outsider, and might perhaps be interrogated or be seen as a potential convert if you profess disbelief. .
As an agnostic atheist raised in a strongly Catholic society (the Philippines), I've had more than enough experience in being a non-Christian visitor in Christian mass. If you really want to see the different Christian traditions, try going instead to larger congregations, pick a mass held in a cathedral if you can. That way you won't have to defend yourself from overzealous conversion. In such environments, you won't have to pretend to be Christian. You won't even be asked. People don't really take much notice of others in large churches, except during the part where they say "peace be with you" to the people around them. The same is true with the older and larger denominations like the Anglican Church or the Orthodox Church. Just try to follow what everybody else is doing (e.g. stand when they stand). However, it's perfectly alright if you don't sing, chant, eat the bread (in fact, you shouldn't), or make the sign of the cross, you can even elect to stay seated during the time when the participants kneel. That's usually what non-Catholic people here do when taking part in a Catholic mass (usually in cases of mixed-religion families). Nobody will glare at you, they only do that when you fall asleep, lol. It's far less uncomfortable than going to evangelical churches or one of those extremely small protestant churches. Any stranger can enter one without having to be confronted with his beliefs, whereas in evangelical churches, you will likely immediately be noticed.
Admittedly, despite being an atheist, I'm more sympathetic to Catholics, as I was raised one, and most of my friends and relatives are Catholics. I know Catholics have a rather unfair image in American culture of being ultratraditionalist and pious due to their adherence to rituals and the "no sex before marriage"/"no divorce"/"no prophylactics" thing, but it's rather the opposite really. They're far less aggressive in converting people (with 1 billion members, they're pretty secure), are more tolerant of other faiths (at least at present times), largely self-sufficient (i.e. they don't trick your grandma into giving them her life savings), their clergy don't have personal net worths in the billions like Pat Robertson, and they're actually less fundamentalist than the Protestant churches that dot rural America. -- OBSIDIANSOUL 22:25, 22 October 2012 (UTC)[reply]
(outdent) OK - this I can answer.
1) The idea of non-denominational ministries is very much associated with Protestantism, especially in America. The reason is simple - one can't be a non-denominational Catholic, Anglican or Orthodox, because membership of those traditions more or less requires adherence to a corresponding denomination. In Europe, there are major churches representing Protestant traditions such as Lutheranism and Presbyterianism - and in some places, those are the state-backed established churches. It's in America that free-floating Protestantism thrives.
2) Evangelicalism is definitely a movement or tradition,and not a denomination. So it's entirely possible for something to be interdenominational and yet thoroughly evangelical. Of course, they might not appear so completely interdenominational if you mentioned you were Roman Catholic, or Orthodox. The idea of a genuinely interdenominational ministry that insists not only on a particular canon of scripture, but also on a specific translation (obscure, and both commissioned and sold by the ministry itself) seems odd to me. I can generally expect to go into a church of my own denomination or any of the other major ones round here, and find several different translations on hand. I might even hear as many as three translations in the course of a single service. Most of those are just that - translations. No interpretation is provided, except perhaps for some tellingly biased chapter headings and a few notes from the translators.
3) Evangelicalism is huge. It's probably the largest single cross-denominational movement in Christianity. So my observations about Living Stream are in no way indicative of all Evangelicals! Indeed, Evangelicals have contributed substantially to the Bible translations used in my own and other denominations, and to their worship styles. I don't think it's possible to make useful generalisations about the trustworthiness of evangelicals.
4) The claim to be cross-denominational is considerably undermined by the fact that there is a close connection between the Local Churches and Living Stream.
5) So I would suggest that Living Stream are not really cross-denominational, or representative of evangelicals generally. I have no way to judge their trustworthiness, except to say that if you put "Recovery Version" into Google, one of its leading auto-complete suggestions is "Bible cult". Our article about the Local Churches movement has some poorly-referenced claims about the specifically Chinese origins of this movement.
6) Please feel free to ask further. I've got an associate's degree in theology (from an institution which does not grade according to doctrinal soundness), so some technical and historical queries may be easier to answer than others. AlexTiefling (talk) 21:44, 22 October 2012 (UTC)[reply]
I'm thinking this may differ from place to place. My experience with the phrase "non-denominational" is that it's often used to deceive potential congregants into seeing a church as more open and more mainstream than it actually is. And they always are a member of a denomination - they just don't want to admit it right off. --NellieBly (talk) 23:00, 22 October 2012 (UTC)[reply]
I think I'd view a claim of being 'non-denominational' as valid if (a) it applied to a Free church which genuinely was not part of any larger structure, or (b) it applied to a publisher, service provider or other organisation which produced material which was applicable to the practice and worship of several denominations. But I broadly agree with your criticism of the way the term is used. I'd also suggest that it's used to appear less mainstream. I often see the term 'mainline protestant' used in the US in a vaguely negative sense to refer to (eg) Lutherans - the implication being that Protestant groups with more divergent eccelsiology (such as Baptists) or fewer ties to 'corrupt' mainstream society. Some of these organisations are themselves both modern and regressive all at once; the Southern Baptists are the only large denomination to have abandoned the ordination of women to the ministry. Amazingly, not all of their existing women ministers walked out. (See thread further up about this topic.) AlexTiefling (talk) 23:09, 22 October 2012 (UTC)[reply]
The internet resource I refer to is Beliefnet, which will give you the answers for questions about Christianity and other religions too. --TammyMoet (talk) 10:15, 23 October 2012 (UTC)[reply]
Well... I would not say beliefnet.com (or any other website) can give the answers to religious questions, but it certainly gives an answer to such questions. Blueboar (talk) 14:51, 23 October 2012 (UTC)[reply]
Indeed. How much bloodshed could have been avoided if certain groups had not insisted they were the one sole true religion. There are many one sole true religions. -- Jack of Oz [Talk] 20:37, 23 October 2012 (UTC)[reply]

Start all over. Frank Mead's Handbook of Denominations is a great introduction to religion in the USA; despite its name, it doesn't restrict itself to Christians, so you won't see groups being left out simply because the author thought that they weren't truly Christians. Granted, it's just in the USA, but so many denominations in other countries are present in the USA that this will be a good introduction to Christianity in other countries. You'll need to buy it or borrow it from a library, as it's new enough that it's not online, but this work is perhaps the best thing that you can get in any way, shape, or form. Nyttend (talk) 02:27, 24 October 2012 (UTC)[reply]

The OP should also understand, if it's not already clear, that "tradition" in theological terms means something a bit different from the everyday, informal use of the word. Nor is there a single, universally agreed definition among various churches. If you re-read the Sacred Tradition article as well as the related Sola scriptura article, you will see that in the theological sense, "tradition" to some people means "a collection of practices and doctrines which accrue, gradually becoming something more developed" - or in other cases, "patristic, conciliar, and liturgical texts" and commentaries on them - or the "historical continuity of interpretation [of the Bible] and teaching" of a certain church. But generally speaking, all that is meant by "sacred tradition" of one church or another is in written form somewhere - basically IMO it boils down to "all the teachings of a church beyond the words of the Bible" - but it's not a big secret that only people who are already members of that church know - it can be looked up and researched for a given church or denomination. Whether it's worth your while to do so is another question, of course. Textorus (talk) 02:25, 25 October 2012 (UTC)[reply]

You can always read Wikipedia. Though, it will probably take some time trying to remember all sorts of religion-related trivia. Certainly, looking up and researching your desired information for a given church or denomination would be useful. However, one may have to keep in mind that the words are descriptory and may not capture the emotions. Perhaps, experience and emotions at a real church service can add to the knowledge about the church. 140.254.227.51 (talk) 18:33, 25 October 2012 (UTC)[reply]

Cycling and drug cheats

Why has this sport been dominated by drug cheats? Ankh.Morpork 21:03, 22 October 2012 (UTC)[reply]

Simply because it is hardly possible to compete without it, and apparently drug tests were inadequate for detecting the drugs used in many cases. Maybe a better question would be why other sports haven't. Or have they? - Lindert (talk) 21:48, 22 October 2012 (UTC)[reply]
That's circular cycular reasoning, Lindert, It's become "hardly possible to compete without drugs", precisely because the playing field has been distorted so badly by people taking drugs in the first place in order to give themselves an unfair advantage over their opponents. It's almost an "if you can't beat 'em, join 'em" situation. -- Jack of Oz [Talk] 22:21, 22 October 2012 (UTC)[reply]
I understand Lindert's argument. One person in sport X uses performance-enhancing drugs, gets away with it, and wins. The losers then realize they must take performance-enhancing drugs if they are to win (even if they don't know that the winner was using them). So, "one bad (undiscovered) apple spoils the bunch". StuRat (talk) 22:35, 22 October 2012 (UTC)[reply]
Yes, it's survival of the fittest. The fittest are the ones that use performance-enhancing drugs. It's only natural that fair players 'die out'. - Lindert (talk) 22:38, 22 October 2012 (UTC)[reply]
Although let's be careful about suggesting inferring that all successful cyclists are abusing drugs. Alansplodge (talk) 01:38, 23 October 2012 (UTC)[reply]
Yes, don't do that, or you will be infer it. StuRat (talk) 05:16, 23 October 2012 (UTC) [reply]
(edit conflict) The answer is, of course "Because they can". For decades, it was ridiculously hard to get caught, you had to be a total idiot in order to get caught. If using PEDs meant you got to make shitloads more money than everyone else that wasn't, AND there was next to no chance of getting caught if you were careful, then it seems easy to understand why. --Jayron32 22:36, 22 October 2012 (UTC)[reply]
(ec)I don't think cycling is the only professional sport dominated by doping, doping occurs in practically all professional sports. It just seems to be the one sport where there has recently been an overall campaign against it in the media as well as in some sports organisations. --Saddhiyama (talk) 22:39, 22 October 2012 (UTC)[reply]
No other sport has experienced the same level of systemic abuse. Since 1996, it has only happened once that all top three cyclists in the Tour de France have tested clean (2012) and it has occurred 13 times that all three, or two of the top three, were found to have used performance enhancing drugs. Lance Armstrong, Floyd Landis, Alberto Contador, Bernhard Kohl, Frank Scleck are examples of recent high-profile cases and it is no exaggeration to state that they represent the norm, not a minority. Ankh.Morpork 08:05, 23 October 2012 (UTC)[reply]
I agree that doping is a problem in many professional sports. I think, at least until recently, endurance athletes have had an easier time getting away with it. The drug of choice in say, sprinting or baseball is an anabolic steroid, whose metabolites are detectable in the urine a lot longer than EPO is, which a cyclist uses. It makes it easier for a cyclist to dodge drug tests: if he can get even short notice of an upcoming test, he can get clean pretty easily. Inject some saline right before blood is drawn for testing. Etc. EPO is also produced naturally by the body, making it harder to tell whether a cyclist has taken the stuff, or is just producing it at higher levels than average. The typical way to tell the difference is different patterns of glycosylation, but it's pretty subtle. Buddy431 (talk) 00:38, 23 October 2012 (UTC)[reply]
For more info how Armstrong was beating testing, as detailed by the recently released report from the USADA, you can look at [4]. There were several factors: off-season drug tests could be pretty easily avoided, the tests weren't that sensitive (or even didn't exist for some of the stuff they were taking), the team learned about "unannounced" drug tests ahead of time, the team was able to mask the signal of drugs with different techniques. That's not to say that similar techniques don't occur in other sports. The steroid THG was nicknamed The clear because (for a while) there was no test for it. This was what was used in the US baseball doping scandal a few years ago, as well as other athletes in other sports. Buddy431 (talk) 01:05, 23 October 2012 (UTC)[reply]
A lot of good points have been made above, but there's also a simple factor. To the best of my knowledge, there is no drug that improves coordination skills in the long term, so it is unlikely that any drug will have much of an effect on golfing ability, for example. In sports like baseball, where coordination and strength play a role, the effect of building muscle will be muted if someone has poor coordination. Hence, in endurance sports, there is just more to gain. IBE (talk) 02:33, 23 October 2012 (UTC)[reply]
Well because initially they (drugs, blood doping, etc) were legal - see blood doping: "Blood doping probably started in the 1970s but was not outlawed until 1986. While it was still legal, it was commonly used by middle and long-distance runners.". After they were outlaw, concerns arise regarding the reliability and competence of some laboratories doing the testing. The World Anti-Doping Agency has also been criticized for not having an effective criteria for testing for EPO. The cycling's governing body might not be trusted to do its own anti-doping, and efforts to develop a coherent anti-doping strategy have been undermined by political in-fighting. See (or listen) to the two part BBC podcast: Secrets in the blood part 1part 2 (First broadcast on 28 Jul 2008, so even back then it's a widely know secret and nothing new.) Royor (talk) 04:19, 23 October 2012 (UTC)[reply]
I've been close to a few sports at top level (no, not as a competitor), and I would suggest that cycling wasn't really trying as hard as some others to find the cheats. Timing of tests was well known, and too far from competition time. And Armstrong did bring in the crowds and money. One sport I'd like to hear more about, but which we don't hear much of in the English speaking world, is cross country ski racing. A fiend gave up 15 years ago because he was too principled to do the drugs all the leaders were doing. As for golf and similar sports, the likely area to find drugs is their use in recovery from injury. Steroids can help a lot there. HiLo48 (talk) 07:28, 23 October 2012 (UTC)[reply]
A quick Google gave lots of entries for Nordic skiing and drugs - for example, the entire Finnish ski team was banned because of drugs, and lots of Russians. It seems that there were lots of bans in the early 00's, but that the number of bans have dropped off since - with the exception of Russians. It seems still to be widespread there. With ski jumping, maybe appetite suppressants would be the drug of choice: Sven Hannawald retired because of the ravages of anorexia, and I'm sure others suffer too. Personally I'd love to know what Ole Einar Bjorndalen was on, I'd love to believe it was fresh air and an addiction to exercise. Let's see what the coming winter brings to this fantastic sport. --TammyMoet (talk) 10:10, 23 October 2012 (UTC)[reply]
Not all sports are equally dependent from endurance alone. Soccer needs some talent, basketball needs some talent, baseball needs some talent. You cannot get those things from a drugs. Comploose (talk) 09:24, 23 October 2012 (UTC)[reply]
True, but I can't see the relevance. HiLo48 (talk) 10:20, 23 October 2012 (UTC)[reply]
Answering the question: the field is dominated by drug cheats because it's possible to dominate the field cheating with drugs. If wannabe soccer/basketball/baseball could cheat through drugs, their whole respective fields would also be dominated by drug cheats, but clean players have good chances to dominate through talent. It's just a summary of things said above. Comploose (talk) 11:18, 23 October 2012 (UTC)[reply]
The fact that the international organization is going to leave all those years' titles vacant does not speak well of the stewardship of the sport by the organization. ←Baseball Bugs What's up, Doc? carrots02:51, 24 October 2012 (UTC)[reply]
In its statement, the UCI said it "acknowledged that a cloud of suspicion would remain hanging over this dark period - but that while this might appear harsh for those who rode clean, they would understand there was little honour to be gained in reallocating places."[5] -- Jack of Oz [Talk] 00:34, 27 October 2012 (UTC)[reply]

Abbreviations: "Ant. ad. Magn."

What do the above abbreviations stand for? Also, it was near an abbreviation "Ern." The "Ern." abbreviation is not a name, but a title. Lastly, there was "Ern. Ant." before a name. It might be French or Latin.

Thanks for your time. Ribbiters (talk) 23:42, 22 October 2012 (UTC)[reply]

It seems it's Latin and stands for "Antiphon ad Magnificat", apparently some kind of hymn. "Ern. Ant." is used as an abbreviation of a name here e.g.: Ern. Ant. Nicolai = Ernst Anton Nicolai. - Lindert (talk) 00:17, 23 October 2012 (UTC)[reply]
The antiphon for the Magnificat is a short prayer or anthem used before the Magnificat itself at Evening Prayer. The most famous examples are the Great Advent Antiphons, used in the week before Christmas Eve - but there are plenty of others. AlexTiefling (talk) 12:00, 24 October 2012 (UTC)[reply]


October 23

Family tree

I apologize if my question is in the wrong Reference Desk subheader, but (as an example) if my mother divorces my father and marries another man, what is my family relationship to that man? Thank you in advance. 71.146.0.234 (talk) 04:58, 23 October 2012 (UTC)[reply]

He would be your stepfather and you would be his stepson. ←Baseball Bugs What's up, Doc? carrots05:06, 23 October 2012 (UTC)[reply]
Thanks. As you can see, I am the equivalent of a tomato when in it comes to family relations. 71.146.0.234 (talk) 05:18, 23 October 2012 (UTC)[reply]
A cross-pollinated tomato or a grafted tomato? —Tamfang (talk) 17:02, 23 October 2012 (UTC)[reply]
And a tomahto or a tomaydo? -- Jack of Oz [Talk] 23:55, 23 October 2012 (UTC) [reply]
Definitely a tomaydo. 71.146.0.234 (talk) 04:49, 24 October 2012 (UTC)[reply]
Cf. Nix v. Hedden. Nyttend (talk) 15:05, 27 October 2012 (UTC)[reply]
... which I've just edited, because it was making wild and unsupported claims about what supposedly the tomato "legally" now is. It needed to be brought back to what the court's ruling was actually about, viz. the customs regulations made under the Tariff of 1883. How people classify the tomaydo in their own minds outside of that context has always been, and remains, their own business. -- Jack of Oz [Talk] 21:19, 27 October 2012 (UTC)[reply]
Probably a cross-pollinated one, but it acts as a grafted one on Sundays. 71.146.0.234 (talk) 04:49, 24 October 2012 (UTC)[reply]
As for whether he would appear on your family tree, that all depends on what you want out of it. If you only list blood relations, then he doesn't belong. However, if he has children with your mother, those half-siblings of yours are blood relations, so you might want to add him back in, to explain where they came from. :-) StuRat (talk) 05:15, 23 October 2012 (UTC)[reply]
Thanks. 71.146.0.234 (talk) 05:18, 23 October 2012 (UTC)[reply]

Stock prices

If people are trading thousands of shares from stock X at price P and then someone pays P + 1 for one single share, will the stock price be now this P + 1? Comploose (talk) 09:22, 23 October 2012 (UTC)[reply]

It depends which type of "stock price" you mean. The last traded price will now be P + 1, so in that sense the answer is yes. On the other hand, if you are tracking an average price for the day (or for a longer period) then a single trade will not change that by much. Gandalf61 (talk) 10:18, 23 October 2012 (UTC)[reply]
What is the reported price in news tickets and such? Comploose (talk) 11:08, 23 October 2012 (UTC)[reply]
Newspapers usually quote the previous day's closing price. The exact definition of "closing price" can be a little complicated. You can see the London Stock Exchange glossary here, but I didn't particularly understand the definition! --Tango (talk) 11:37, 23 October 2012 (UTC)[reply]
Once again, it depends. The Google ticker shown here is displaying the last traded price. However, in a quote driven market you might see the mid price which is the average of the lowest offer and highest bid prices currently quoted in the market - there might not be any actual trades executed at this price. Gandalf61 (talk) 11:54, 23 October 2012 (UTC)[reply]

Why didn't Israel respond to Iraqi attacks during the War???

Thank you. Iowafromiowa (talk) 11:05, 23 October 2012 (UTC)[reply]

The US asked Israel for friendly restrain, for not setting the whole area in fire. Since Saddam Hussein's strategy was to drag Israel into the conflict, it was perfectly reasonable to do the opposite. Comploose (talk) 11:13, 23 October 2012 (UTC)[reply]
Yeah, this is pretty much the main reason. Futurist110 (talk) 22:10, 23 October 2012 (UTC)[reply]

See Gulf_War#Iraq_launches_missile_strikes. The US was nominally part of a Coalition of forces, which included many Arab states. It was felt that Israeli retaliation would jeopardise the Coalition. --Dweller (talk) 12:08, 23 October 2012 (UTC)[reply]

A good deal of Coalition effort was expended in trying to prevent Iraqi Scud missiles being fired at Israel; thereby hoping to prevent domestic political pressure from forcing Israeli retaliation. Special forces surveillance combined with air strikes in the launch areas were effective, but not entirely so.[6] The success of the Patriot missiles that the US sent to defend Israel is still being debated.[7] Alansplodge (talk) 12:13, 23 October 2012 (UTC)[reply]
Actually, the missiles were a great success. Admittedly, it was a political success. The operational success is indeed very much up to debate... --Stephan Schulz (talk) 14:28, 23 October 2012 (UTC)[reply]

How much does interest cost the Greek Government?

Greece has tons of national debt at the moment, how much (annually, ideally) is it costing to pay the interest on? What % is that of GDP? And finally, I've seen on a couple of sites that 'experts' say that once you are paying 12% of GDP in interest that is a historical tipping point for default. Are there any good references for that? Many thanks, 46.30.55.66 (talk) 14:18, 23 October 2012 (UTC)[reply]

In 2011, interest payments by the government (only) were €15 billion, or 7.1% of GDP. Source: [[8]]. Bear in mind that this does not include principle, short-term lending or debt service by the private sector. DOR (HK) (talk) 06:01, 24 October 2012 (UTC)[reply]
Edit: should be "short term debt repayment, including both principle and interest," not "short term lending.DOR (HK) (talk) 06:03, 24 October 2012 (UTC)[reply]
Thanks a lot, that's great. (OP on a different IP) 86.166.191.232 (talk) 08:13, 24 October 2012 (UTC)[reply]

What games do Boy Scouts of America play that Scouts in England do not?

What games do Boy Scouts of America play that Scouts in England do not? A Scout in my troop here in England needs to teach Cub Scouts an American Scouts game that is not played in England, in order to earn his Global Challenge badge. 82.31.133.165 (talk) 14:56, 23 October 2012 (UTC)[reply]

I do not know myself (I was a scout for about a year in my early teens and rose to the high rank of Tenderfoot) but surely there are materials online that would provide better proof than a claim by some random on a Wikipedia forum?--Wehwalt (talk) 15:00, 23 October 2012 (UTC)[reply]

Online I can find some games from England and perhaps some games from America, but from these websites I am uncertain if any of the games are common only in America and not England. 82.31.133.165 (talk) 15:23, 23 October 2012 (UTC)[reply]

I realise I'm about to completely ignore your question, but please bear with me. A better way to go about this might be to support the Cubs in finding out for themselves about games in US scouting. Learning by doing, and all that. I would suggest speaking to your District or County International Adviser for support in making contact with a group in the US, and then asking them about their weekly programme. If you can't get hold of your International Asviser, try ringing Gilwell's Information Centre on 0845 300 1818 or 0208 433 7100 - they are incredibly helpful and knowledgable. Alternatively, if none of these things are possible, try getting the Cubs to do some research themselves - http://www.scouting.org would be a good place to start. - Cucumber Mike (talk) 15:31, 23 October 2012 (UTC)[reply]
(ec)The best practice on the Ref Desk has always been to link to reliable sources in answering a question. In the USA, there is Boys' Life magazine, which might lists games that Scouts play. Many issues over a period of many years are available for reading online via Google Books, such as this one from 2009. Sadly, the "games" section on page 50 only talks about videogames, not the sort where you run around playing hide and seek or whatever. The handbooks for the Boy Scout and Cub Scout organizations include games, but I no longer have any of those handy for reference in my home. An online source for games played by scouts in the US is "Boy Scout Trail". Edison (talk) 15:35, 23 October 2012 (UTC)[reply]
Well... I know that American Scouts play pick-up games of Baseball from time to time... unlikely in the UK. Blueboar (talk) 15:44, 23 October 2012 (UTC)[reply]
Kickball is even easier to put together than baseball: same basic rules, less equipment necessary. --Jayron32 16:41, 23 October 2012 (UTC)[reply]
One game they've at least played in the recent past has to do with a literal application of the old joke about starting a fire "by rubbing two boy scouts together." ←Baseball Bugs What's up, Doc? carrots02:55, 24 October 2012 (UTC)[reply]
Hilarious. You should take your act on tour. AlexTiefling (talk) 09:39, 24 October 2012 (UTC)[reply]
Well, there might be child-molesting going on the UK scouts too, but the US situation is what got publicity recently. ←Baseball Bugs What's up, Doc? carrots12:43, 24 October 2012 (UTC)[reply]
The Scout Association of the United Kingdom has been at the forefront of the development of robust child protection procedures, but no system is entirely infallible. My opinion is that jokes aren't an appropriate response to this issue, moreover, I fail to see how it is connected with the original question. Alansplodge (talk) 19:03, 24 October 2012 (UTC)[reply]
Dark humor. And the joke about rubbing two scouts together was probably innocent, as I'm fairly certain it was on TV at a time when vulgar stuff wasn't allowed on American TV. — Preceding unsigned comment added by Baseball Bugs (talkcontribs) 12:11, 25 October 2012 (UTC)[reply]
Well maybe. But when our voluntary youth worker posted his question above in good faith, he was probably hoping to avoid a load of innuendo about child abuse. That is all I have to say. Alansplodge (talk) 12:23, 25 October 2012 (UTC)[reply]
I admit to being highly suspicious of both the Catholic church and the scouts as institutions. As to the OP's question, I would think baseball and/or softball would be obvious answers, as those games are not likely to be commonly played in the UK. Pitching horseshoes also comes to mind. ←Baseball Bugs What's up, Doc? carrots12:28, 25 October 2012 (UTC)[reply]

Back to the point; try MacScouter's Great Games Resource which lists 120 "Games for Younger Scouts" (it has a picture of Cub Scouts on the title page). Good hunting. Alansplodge (talk) 19:15, 24 October 2012 (UTC)[reply]

Obvious answers: baseball, American football.

New Mexico vs. Arizona

New Mexico is currently projected to go solid Democrat, while Arizona is solid Republican. What accounts for the difference? Arizona is several times larger in population than New Mexico and its economy is also several times larger; New Mexico seems more dependent on government funding (lots of military bases, two national labs) than Arizona. Arizona gets in the news for its harsh immigration policies; New Mexico, one would suspect, would have similar issues. New Mexico seems unique among states along the southern border of the US in its political affiliations (California excluded but California is its own weird thing). Why is New Mexico a bastion of Democratic support? --Mr.98 (talk) 15:17, 23 October 2012 (UTC)[reply]

I think you hit the nail on the head with your observation that in New Mexico, the military is a major employer. The military tends to disproportionately employ the young and minorities, both of which tend to vote more Democratic. Also, minor in comparison, the city of Taos, New Mexico is a bit of an artist colony (see Taos,_New_Mexico#Taos_art_colony), and you know how those crazy beatniks vote ! StuRat (talk) 15:29, 23 October 2012 (UTC)[reply]
Yeah, I think that the larger percentages of Latinos/Hispanics and Native Americans in New Mexico are the main things that make it more Democratic than Arizona. Futurist110 (talk) 22:08, 23 October 2012 (UTC)[reply]
Fivethirtyeight.com did a pair of articles on the two a while ago: Arizona; New Mexico. NW (Talk) 19:28, 23 October 2012 (UTC)[reply]
It was always so, at least post-1848. Arizona has always had more of an Anglo population than NM. That is, basically why they did not enter as one state, as Teddy Roosevelt wanted.--Wehwalt (talk) 19:38, 23 October 2012 (UTC)[reply]
Also, New Mexico is over 9% American Indian, while Arizona is about half that. Angr (talk) 20:59, 23 October 2012 (UTC)[reply]

Mr.98 -- In northern New Mexico there's a large grouping of Hispanics who are all U.S. citizens, and who are very conscious that they are not recent immigrants, but in fact were there before the "Anglos" were... AnonMoos (talk) 23:46, 23 October 2012 (UTC)[reply]

Winner-Take-All

Considering all states (except the two weird ones :-) ), is there a reason why states adopted a winner-take-all system when casting its electoral votes for a presidential candidate? I can understand the disadvantages of the system, the wiki article on that is quite good. But nowhere (including wikipedia and google) can I find the reason almost all of the states would adopt such a system? At least what would be the (political science) theory behind adopting such a system? Does it give one party an advantage over another? Also I can't find its history anywhere either. Has this been in place since the constitution or was it adopted slowly over the last two centuries? Did states gradually implement it or did they all do it at once? States are given almost complete power over their local election process so we have this huge variety of different rules. Winner-take-all is the only rule which seems almost universal but it isn't federally mandated like voting age being 18. I can't imagine all of the states unanimously agreeing on something out of their own free will. Just curious what led to it. It also seems to be the reason why we have had two party system for more than a century no to mention gives the media too much power and influence.70.58.0.141 (talk) 22:22, 23 October 2012 (UTC)[reply]

We did this a few weeks ago. It's to the advantage of a state, in a certain sense, because it makes it more likely that the person who is elected will be the plurality choice of the voters in that state. This is especially true for large states — in an election that's close in all states, an individual in a large state actually has more chance of flipping the election as a whole than a voter in a small state (whereas without winner-take-all it would be the reverse).
The downside is that if the election isn't close, the state gets ignored. Wait, did I say downside? --Trovatore (talk) 22:29, 23 October 2012 (UTC)[reply]
States that lack the winner-take-all rule are virtually ignored by the candidates, since there are very few electoral votes in play. If the candidate worked like hell there, maybe he could flip the vote from 45% in his favor to 55%, but that would only amount to a 10% difference in the number of state electors he got (which might not be any actual electors, if the state has less than 10). Compare this to a winner-take-all state where a much smaller effort, say one that flips his support from 49% to 51%, would gain him 100% of that state's electors (and deny them to his opponent). StuRat (talk) 03:27, 24 October 2012 (UTC)[reply]
Yes, winner-take-all makes candidates pay much more attention to large swing states. However, large states that are fairly safe for one party, like California or Texas, would probably get more attention if they dropped winner-take-all. Right now everyone knows Obama will get all of California's votes. If California went proportional, Obama would still get the majority of them, but the candidates would have to address California-specific interests in an attempt to increase or reduce Obama's margin.
However, the party in power in such states has to balance that against the fact that dropping winner-take-all would improve the other party's chance of winning the presidency. --Trovatore (talk) 04:07, 24 October 2012 (UTC)[reply]
I believe Republican strength still lies in Orange County, various areas in the California Central Valley, and in some regions in the eastern part of the state. So, yes, if California ever went proportional, it would be more than likely than some electoral votes would instead go to the Republican candidate. Zzyzx11 (talk) 04:58, 24 October 2012 (UTC)[reply]
The red states already have a disproportionate advantage in the electoral college, and turning a state like California into "proportional" electoral voting would ensure that a Republican gets elected President every time. I'm sure Republicans would like that. But you can't do that. The people don't elect the President, the states do. If they want a true proportional vote, then the electoral college needs to be abolished in favor of a direct popular-vote election. Don't hold your breath waiting for that to happen. ←Baseball Bugs What's up, Doc? carrots05:10, 24 October 2012 (UTC)[reply]
Lord, please let this not become a political debate. At least, not here.
Bugs, this isn't the place to be advocating any kind of political changes, and you know that. -- Jack of Oz [Talk] 05:26, 24 October 2012 (UTC)[reply]
I'm not advocating any change. I'm explaining the consequences of such a change. ←Baseball Bugs What's up, Doc? carrots05:55, 24 October 2012 (UTC)[reply]
Sure you are. I believe you. Just, remind me: What was that you were saying below about facts vs. spin? -- Jack of Oz [Talk] 06:46, 24 October 2012 (UTC)[reply]
Looking at the math a little closer, I see that my blanket statement was incorrect, but much depends on how the electoral votes are allocated. I took just the 2008 results and figured out the electoral vote proportioned in two different ways. The original electoral vote was 365 to 173. If you split each state's electoral votes to two decimal places, it comes out 283.14 to 246.45, with 8.41 for the miscellaneous candidates. That matches the popular vote proportion exactly. So Obama still wins, and it appears to nullify the red state vs. blue state effect. If you round within each state, you get Obama with 280, McCain with 243, and 16 missing due to the rounding. And that raises the question of what's going on with Nebraska, the one state that split its electoral vote in 2008. 5 electoral votes, 4 given to McCain and 1 to Obama. But based on popular vote proportions in the state, it should have been 3 and 2. ←Baseball Bugs What's up, Doc? carrots07:29, 24 October 2012 (UTC)[reply]
It turns out that they create "districts" within the states, and it's clear that a state could gerrymander around the theoretical need to apportion its popular vote. ←Baseball Bugs What's up, Doc? carrots07:33, 24 October 2012 (UTC)[reply]
States that want to go proportional can do it any way they want to. They have almost plenary power to appoint their own electors. --Trovatore (talk) 07:55, 24 October 2012 (UTC)[reply]
Yes they do, currently. If there is eventually a clamor for a direct election, obviously that would require a constitutional amendment and would nullify the red-states' advantage - which is why such an amendment will never pass. ←Baseball Bugs What's up, Doc? carrots12:30, 24 October 2012 (UTC)[reply]
This supposed "red-state advantage" is actually not clear at all. In 2000, a change of a small number of votes in Florida and Ohio, combined with a slightly better Bush performance in a few blue states, could have easily resulted in Bush winning the popular vote but Gore winning the election. In recent weeks CNN has been showing the popular vote between Obama and Romney as all tied up within the margin of error, but still has been giving a clear electoral advantage to the president, so an entirely plausible outcome of this election is that Romney will win the popular vote but Obama will be re-elected. --Trovatore (talk) 22:37, 24 October 2012 (UTC)[reply]
There's a move going on, slowly, that may eventually render the electoral college obsolete. There is a growing number of states who are making agreements with other states to commit their electors to whoever wins the national popular vote. This agreement won't take effect until at least 270 electoral votes worth of states enter the agreement, and they are only about halfway there yet. Unless a constitutional challenge is mounted succesfully, this approach might not only render the electoral college moot, it could revolutionize the approach to creating amendments. You could call it a pseudo-amendment. ←Baseball Bugs What's up, Doc? carrots01:27, 25 October 2012 (UTC)[reply]
I don't see how it could "revolutionize the approach to creating amendments" in general. The scheme relies on the idea that states can appoint their electors any way they want, and can therefore decide to give them to the winner of the national popular vote, if they want. That is probably true, but it seems very specific to this one issue. Most things that you'd want to accomplish by an amendment can't be implemented by just a certain number of states deciding to do it. --Trovatore (talk) 02:04, 25 October 2012 (UTC)[reply]
Well, one thing I could see would be in the area of abortion rights. Since the GOP will never get the 2/3 majority needed in the US House and Senate, and might or might not be able to get a right-wing activist court to overturn Roe v. Wade, they might take their strategy to the states and create a pseudo-amendment. ←Baseball Bugs What's up, Doc? carrots12:09, 25 October 2012 (UTC)[reply]
I don't see how they could do that. The electoral vote scheme doesn't override anything in the constitution -- it's just a way of collaborating on how to vote. Specifically what could a group of anti-abortion states agree to do that would have the effect of allowing them to ban abortion? Duoduoduo (talk) 16:14, 25 October 2012 (UTC)[reply]
It's the National Popular Vote Interstate Compact... AnonMoos (talk) 12:30, 25 October 2012 (UTC)[reply]
  • Part of the benefit of the electoral college system is that it requires candidates to focus on winning a broader area instead of just a higher vote. The nine most populous states List of U.S. states and territories by population outweigh the population of the other 41. But those 41 have an advantage of 64 electoral votes (82-18=64) over the top nine in the electoral college system. Hamilton was an elitist, and his argument that electors were more qualified than the general populce no doubt appealed to him. But I doubt his rationale was the real reason why the small states were happy with the compromise. μηδείς (talk) 17:12, 25 October 2012 (UTC)[reply]

Early Voting

A different question, why do states implement early voting? And it also looks to me like as if various states can be quite aggressive in promoting it too as if they prefer/want me to vote early. I want to wait until I make my decision so why would I vote early? Is it just so that the lines are shorter on the general election day? It doesn't sound like a good enough reason when you see how much emphasis is put on voting early. Do all states wait until the general election day to count the early votes or do some states start releasing info before November? Because if counting is done and announced before November 6 then it has the potential to drastically change the other voters' minds and swing the election.70.58.0.141 (talk) 22:22, 23 October 2012 (UTC)[reply]

Have you read Early voting, Absentee ballot, and Postal voting? As far as I can tell, such votes are never counted until polls are closed on voting day, precisely to avoid influencing the election. I've heard rumors that absentee ballots are often not counted until days or weeks later, which means (in the case of a U.S. presidential election) they don't get counted until the state has already been called for one or the other candidate. Presumably these rumors only apply to states without no-excuse early voting, as it wouldn't make sense to allow and even encourage your voters to vote early if you're not going to count the votes until it's too late for them to make a difference anyway. Angr (talk) 22:38, 23 October 2012 (UTC)[reply]
There's no "too late for them to make a difference". If the election has "already been called" in that state, it means that it wasn't close enough for the absentee ballots to change the result, and therefore they would never have made a difference, no matter when counted. If it's close enough for them to matter, you're usually in recount territory, and then they will definitely go into the mix. --Trovatore (talk) 22:41, 23 October 2012 (UTC)[reply]
I expressed myself badly. I knew what I meant. Angr (talk) 22:59, 23 October 2012 (UTC)[reply]
It might be many things but the first thing that comes to my mind is the number of volunteers that the town/district expects to be able to recruit for election day. If there are enough voters who come in and vote early, that means that the town potentially has to find fewer volunteers. In a small town like mine, it's nothing for a few people to come in every day during election day and cast their vote. It likely doesn't add that much work to the town hall staff's day. And then there's the added benefit of the small parking lot at my polling place (our volunteer fire department) not filling up with cars and lines going out the door.
Also, I can take my ballot home, look at the names, research each one of the less important races, and make a more informed decision. Dismas|(talk) 00:04, 24 October 2012 (UTC)[reply]


I have a very strong suspicion as to the reason California, specifically, promotes it so hard. Not in every case, but as a statistical average, Republicans are more likely to vote than Democrats. However, Democrats control the legislature, and while Republicans don't do very well here electorally, the Democrats would like them to do even worse. If they can increase turnout, they will increase Democratic percentages, because the marginally-committed voters they pick up are disproportionately Democratic-leaning. --Trovatore (talk) 00:12, 24 October 2012 (UTC)[reply]
Hence the various national efforts by Republicans to disenfranchise voters who might lean Democratic. ←Baseball Bugs What's up, Doc? carrots02:44, 24 October 2012 (UTC)[reply]
That's the other spin, yes. --Trovatore (talk) 02:49, 24 October 2012 (UTC)[reply]
It's not "spin". The facts speak for themselves. ←Baseball Bugs What's up, Doc? carrots02:53, 24 October 2012 (UTC)[reply]
It is spin. Everyone has a chance to vote; no one is excluded if he wants to vote badly enough. The question is, how hard do we want to try to get people to vote who don't want to that badly? --Trovatore (talk) 02:59, 24 October 2012 (UTC)[reply]
It sounds like BB is taking about things like voter ID cards etc. That being the case, whether you want to say 'how hard do we want to try to get people to vote who don't want to that badly' or 'how much do we want discourage people from voting who don't want to that badly' are both ultimately POV. Nil Einne (talk) 04:33, 24 October 2012 (UTC)[reply]
Voting is a right, not a privilege. The Republicans' efforts have been to make this right as tedious and difficult as possible for certain classes of people, and many of those attempts are being tossed by the courts who see them for what they are: attempts to discourage Democrats from voting. That's not spin, it's fact. ←Baseball Bugs What's up, Doc? carrots05:05, 24 October 2012 (UTC)[reply]
Can you show us a link? -- Jack of Oz [Talk] 05:23, 24 October 2012 (UTC)[reply]
After Trovatore does. :) ←Baseball Bugs What's up, Doc? carrots05:54, 24 October 2012 (UTC)[reply]
How 'bout this one? --Trovatore (talk) 07:58, 24 October 2012 (UTC)[reply]
Very good. Meanwhile, you may find Voter ID Laws in the United States somewhat enlightening. ←Baseball Bugs What's up, Doc? carrots12:36, 24 October 2012 (UTC)[reply]

Promenades en Océanie: les Tubuaï et l'archipel de Cook

Does anybody know when the events written down by M. Aylic Marin in Promenades en Océanie: les Tubuaï et l'archipel de Cook occurred? It was published in 1885, but that would be impossible since the figures mentioned in it died in the 1870s. I need the year he went to those islands and even exact dates if possible. I translated some of it here. Also is M an abbreviation of his first name or Monsieur? And who was he?--KAVEBEAR (talk) 22:51, 23 October 2012 (UTC)[reply]

I'm surprised you didn't try googling for "Aylic Marin". The first link it takes you to is http://fr.wikipedia.org/wiki/%C3%89douard_Petit_%28%C3%A9crivain%29, which answers most of your questions. Looie496 (talk) 23:39, 23 October 2012 (UTC)[reply]
So that narrow the visit between 1880 and 1885. Can somebody find a more specific year?--KAVEBEAR (talk) 23:48, 23 October 2012 (UTC)[reply]
Does anybody know what year Edouard Petit visited the Austral Islands?--KAVEBEAR (talk) 18:17, 26 October 2012 (UTC)[reply]

Rurutu and Rimatara

Was the people of Rurutu and Rimatara converted into the Calvinism denomination or the Wesleyan denomination? --KAVEBEAR (talk) 23:01, 23 October 2012 (UTC)[reply]

Well, Rimatara was converted by two native missionaries dropped off by Samuel Pinder Henry, who was the son of William Henry, who was an Anglican missionary of the low church sort. So most likely Anglican, but possibly slanting in the Calvinist direction. Looie496 (talk) 23:32, 23 October 2012 (UTC)[reply]


October 24

Priest vs. Clergy

Are there more priest in prison for child offenses than other types of clergy, say Baptist or Lutherans? I am not singeling out anyone, I really am interested in the percentages. Many thanks, JeffJDLane13 (talk) 01:20, 24 October 2012 (UTC)[reply]

When you say "priest", are you referring to Catholic clergy? -- Jack of Oz [Talk] 01:21, 24 October 2012 (UTC)[reply]

Yes I am and thanksJDLane13 (talk) 01:45, 24 October 2012 (UTC)[reply]

Define "more". Total per denomination? Or percentage of the given denomination's clergy? ←Baseball Bugs What's up, Doc? carrots02:41, 24 October 2012 (UTC)[reply]
Also: in one particular country, or worldwide? Marnanel (talk) 04:58, 24 October 2012 (UTC)[reply]
And bear in mind that there are 1.2 billion Roman Catholics, and only 75 million Lutherans.--Shantavira|feed me 07:27, 24 October 2012 (UTC)[reply]
The OP did say he's interested in the percentages, which I suppose means looking at the relative proportion of offenders within each denomination. The Catholics do seem to get most of the bad press in this area. That's partly because there are a lot more Catholic priests and brothers than religious of other denominations, and it may be that some much smaller group has a higher proportion of offenders within their ranks than the Catholics do. But I hope that's never going to be used to downplay or point-score, along the lines of "We Catholics only have 15% of our people up on child molestation charges worldwide. You Discalced Restricted Calathumpians of the Blessed Foreskin of Zebedee have 25%, and you're the folks who obviously need to clean up your act before pointing the finger at us". -- Jack of Oz [Talk] 11:19, 24 October 2012 (UTC)[reply]
Overall, I expect you would find the percentage of convicted felons in any denomination is an extremely low percentage. You occasionally hear about child-molesting charges in Protestant denominations. It seems to be same-sex among the Catholic and opposite-sex among the Protestant, which is not necessarily an endorsement or a condemnation of either denomination. ←Baseball Bugs What's up, Doc? carrots12:41, 24 October 2012 (UTC)[reply]
Not necessarily an endorsement? You mean it could be a plus, either way? And not necessarily a bad thing, either way? Regardless of who commits them, are you actually defending these atrocities?
Here's what the Deputy Police Commissioner of Victoria said last week to an official enquiry being held by the Victorian Parliament. -- Jack of Oz [Talk] 21:06, 24 October 2012 (UTC)[reply]
I think Bugs merely meant that there was no moral lesson to be drawn from the apparent gender biases of child abusers from different denominations, which is certainly true. However, I'm not convinced that such biases are statistically significant, and anecdotal evidence has suggested that women and girls abused by celibate clergy have a harder time being believed, because of the alleged same-sex preference factor. Moreover, Bugs seemed to find the topic amusing enough when he joked about it in the Boy Scout thread further back - in response to a question apparently asked by a Scout, no less. AlexTiefling (talk) 07:44, 25 October 2012 (UTC)[reply]
I call it "dark humor". The common thread among these institutions is their public condemnation of "non-conforming" adult sexual interactions, while also tolerating and hiding predatory sexual interactions. As for the scouts specifically, the joke about starting a fire by rubbing two scouts together I first heard several decades ago, probably on TV, and I don't think it had any nefarious implications, although I was really too young to know about such things. ←Baseball Bugs What's up, Doc? carrots11:55, 25 October 2012 (UTC)[reply]
The title of this section keeps bugging me. Priests are clergy; there are clergy who are not priests; there are priests who are not Roman Catholic. AlexTiefling (talk) 11:49, 24 October 2012 (UTC)[reply]
Yes. It's a bit like saying "Pastors vs. Ministers". ←Baseball Bugs What's up, Doc? carrots12:37, 24 October 2012 (UTC)[reply]

The question is comparing apples and oranges. Baptist and Lutheran ministers may marry. You'd have to compare among all unmarried or all married ministers. μηδείς (talk) 16:04, 24 October 2012 (UTC)[reply]

I don't get you Medeis. Married men are just as likely to be child-abusers as are single men, possibly more so as they have more opportunity. DuncanHill (talk) 19:25, 24 October 2012 (UTC)[reply]
You don't get me? More like you are not familiar with how to set up an objective experiment. One compares like to like and controls for independent variables so far as possible. μηδείς (talk) 05:00, 25 October 2012 (UTC)[reply]
wouldn't the later be part of the point though? Nil Einne (talk) 04:31, 25 October 2012 (UTC)[reply]
You could do both. Set it up as an observational study and use multivariate analysis to see if unmarked Catholic priests were more likely to abuse than other clergy in general and if they were more likely than other unmarked clergy You could even see what effect adjusting for the relative density of clergy has. Itsmejudith (talk) 07:11, 25 October 2012 (UTC)[reply]
That's what you might do in order to discover whether or not there's any correlation between marital status and propensity to sexually offend. But Medeis seems to be taking this as a given. Until otherwise demonstrated, the marital status of sex offenders is about as relevant as the colour of their hair or their astrological sign. Unless it's being suggesting that men abuse children or rape people only because they're sexually frustrated as a result of not being married. If so, where's the proof? And what about people who are not married in the eyes of any church or the law, but have ready access to one or more adult sexual partners? -- Jack of Oz [Talk] 08:06, 25 October 2012 (UTC)[reply]
I googled [list of clergy convicted of sexual abuse] just to see what might turn up, and there is no shortage of such lists. This one may be of some interest. The OP could add up the counts given for first letter of last name and divide by the total number of priests to get some very rough idea of a percentage. I googled [number of catholic priests in usa] and one of the items that turned up was Catholic Church in the United States, which suggests America has 40,000+ priests. One caution would be that 40,000 is only the active number of priests, so the ratio of suspects to priests would be inflated, but it would give an upper bound. It's possible someone has done the math on both Catholics and other denominations, which would require further googling. ←Baseball Bugs What's up, Doc? carrots12:06, 25 October 2012 (UTC)[reply]

Tax cuts

If we are to offer people in the U.S. tax cuts with the premise of helping American achieve jobs, innovation and etc., can it be mandated that the money offered must remain in the U.S.? Why would give tax breaks for someone to take it out of the the offering country to another because the make more profit? Wouldn't this defeat the intended purpose? Thanks JDJDLane13 (talk) 01:28, 24 October 2012 (UTC)[reply]

The Reference desks are intended for questions that can be answered with facts or references. That doesn't hold for this question, so it really isn't appropriate here. Looie496 (talk) 01:52, 24 October 2012 (UTC)[reply]
The first question is appropriate for the ref desk. The second and third are the problem. Someguy1221 (talk) 02:01, 24 October 2012 (UTC)[reply]
How exactly would you enforce such a law ? It wouldn't be enforceable on an individual level like that. However, multinational corporations are required to report where they spend their money, so it might work on that level. StuRat (talk) 03:19, 24 October 2012 (UTC)[reply]
Money is fungible... the multinational would simply take some of the funds it was originally planning on spending in the US and spend it elsewhere instead. Blueboar (talk) 03:44, 24 October 2012 (UTC)[reply]
On a technical level, the refund could be in the form of a debit card that is only authorized for purchases from American corporations, although that doesn't guarantee the money will stay in the US in the end. Don't food stamps now come in the form of a debit card that only works at places that sell groceries? Someguy1221 (talk) 03:29, 24 October 2012 (UTC)[reply]
I believe food stamps were always restricted to certain foods. They now have the bridge card, with similar restrictions. Also, a tax cut isn't the same as a tax refund. If the government never has the money, it can't really control what people do with it. StuRat (talk) 03:35, 24 October 2012 (UTC)[reply]
The premise of the question is that keeping the money in the US is best, but that's not at all clear. Suppose that I got a tax cut of $20,000, and I used the money to buy $15,000 of Vietnamese blue jeans with fancy custom stitching on the pockets, then spent $2,000 to have them shipped here and to have, I don't know, $1,000 worth of rivets from Mexico riveted onto the legs in downtown Los Angeles, and then I spent $2,000 on web banner ads on a Canadian website, and I sold half of the jeans here in the US for $30,000, and half in Canada for $20,000. My profit is $30,000, all of which I then use to buy a Honda Accord that was assembled in Ohio. And I use the $20,000 left over to do this again with the blue jeans. Is this so bad? An American (me) now has a $30,000 car plus this interesting cash engine of a blue jeans business. Globalization is super-complicated and although there is an obvious appeal to spending money within the US, because it adds to the country's GDP, it might be better for numerous Americans in the chain if the money goes outside of the US in exchange for something else. Tarcil (talk) 03:56, 24 October 2012 (UTC)[reply]

People are talking about "money offered" and "refunds", which is not what tax cuts are about. The govt would simply be taking less off you than they would otherwise have taken. They're not giving you anything. You're still giving money to them, but less of it. -- Jack of Oz [Talk] 05:21, 24 October 2012 (UTC)[reply]

The idea of Tax expenditures is well-accepted in some contexts... AnonMoos (talk) 06:09, 25 October 2012 (UTC)[reply]
They might be giving you something. Tax credits fall into two classes: refundable and non-refundable. The difference is that only refundable tax credits can bring your tax burden below zero. (An example of a refundable tax credit is the EIC in the US.) If you have enough refundable tax credits on your income tax, you will receive money from the government at the end of the year. They are commonly used as a subtle method of wealth distribution to ensure the poor don't actually starve, while not riling the right wing about "handouts". Marnanel (talk) 06:42, 25 October 2012 (UTC)[reply]

Multiple manslaughter

Is "multiple manslaughter" really a crime in Italian law, or does the phrase simply mean "multiple counts of manslaughter"? Our article on the 2009 L'Aquila earthquake speaks of scientists being convicted of "multiple manslaughter", as does the BBC, so I'm rather confused. Nyttend (talk) 02:29, 24 October 2012 (UTC)[reply]

The news stories I've seen in the Italian press say omicidio colposo plurimo, "plural manslaughter". I don't know whether that's a legal phrase in Italian or not, but at least the Beeb came by it honestly. --Trovatore (talk) 03:02, 24 October 2012 (UTC)[reply]

Mayor of Rurutu

Does anybody know the names and terms of the Mayors of Rurutu? I need to know the name of the mayor in 1982.--KAVEBEAR (talk) 05:41, 24 October 2012 (UTC)[reply]

According to Teuruarii IV's article, his grandson, Toromona (Solomon) Teuruarii, was "mayor of Rurutu in the 1970s". The JOURNAL OFFICIEL DE LA POLYNESIE FRANGA1SE (sic) lists "Solomona Teuruarii" as le maire on 8 May 1980. Clarityfiend (talk) 07:10, 24 October 2012 (UTC)[reply]

Pierre De Chevigne

Was Pierre De Chevigne Secretary of Defence in the French Government in November 1952 because an account in the National Archives,ref. 751G.00/11-2452 of RG 59 of 24 November, has him visiting Indochina for three weeks using that title. You have withthat title only from May to June in 1958.124.176.54.133 (talk) 07:38, 24 October 2012 (UTC)[reply]

The French wikipedia has a detailed list of Chevigné's various mandates [9]. He was Secretary of State for War ("Secrétaire d'État à la Guerre") from August 1951 to June 1954, keeping the title under five successive Prime Ministers in those thays of short-lived governements. The title means he was a junior Minister of Defense responsible for the Army; in those days, France has a Minister of National Defense, an Associate Minister of National Defense (at times), and Secretaries of state for War, for the Navy and for the Air Force. He was briefly the actual Minister of Defense from May-June 1958, in the short-lived Pflimlim Government that was the last government of the IVth Republic. In November of 1952, René Pleven was the Minister of National Defense, and Chevigné was one of three Secretaries of State reporting to him. See here [10]. --Xuxl (talk) 08:34, 24 October 2012 (UTC)[reply]

Electoral College - Tie Scenarios

I am sure they exist somewhere, but I can't to seem to find them. Where can I find a list of possibilities for an Obama/Romney tie? or even colored maps showing such scenarios? Also, what is the most likely path towards a tie come election day? Hisham1987 (talk) 08:17, 24 October 2012 (UTC)[reply]

This has been discussed a bit on FiveThirtyEight.com. The entry on 1st October 2012 discusses it in detail. 86.166.191.232 (talk) 09:43, 24 October 2012 (UTC)[reply]
See Electoral College (United States)#Contingent presidential election by House and Twelfth Amendment to the United States Constitution. In any case where someone doesn't receive a clear majority (more than half) of the electoral votes, then the election of the President goes to the House of Representatives, and the election of the VP goes to the senate, with each state's delegation (not individual legislators, the delegation as a whole) getting one vote. A clear majority (not merely a plurality) is needed for a win: it can be a perfect tie, or it can be a case where there are three candidates receiving votes, and none gets more than 1/2. Per the text of the twelfth amendment "if no person have such majority, then from the persons having the highest numbers not exceeding three on the list of those voted for as President, the House of Representatives shall choose immediately, by ballot, the President. But in choosing the President, the votes shall be taken by states, the representation from each state having one vote" --Jayron32 12:03, 24 October 2012 (UTC)[reply]
Note that the Senate would choose the vice president from among the top two electoral vote recipients for vice president, a majority being necessary. This would be the newly-elected House and Senate, by the way, as they do not meet for the joint session at which the electoral vote is counted until after January 3. Not sure if the vice presidential tiebreaking vote applies, if it goes 50-50.--Wehwalt (talk) 12:12, 24 October 2012 (UTC)[reply]
I think Jayron's and Wehwalt's posts miss the point of the OP's question, which I believe is what plausible combinations of states going for Romney and states going for Obama would lead to a tie. Interesting question. I can't see how to get to the 1 October article on FiveThirtyEight.com that was referenced by 86.166.191.232. Duoduoduo (talk) 14:28, 24 October 2012 (UTC)[reply]
It's here [11]. It's easy to get to by scrolling to the bottom of the blog and following the link to older enteries and keep going back until you find the enteries for 1 October 2012 and then finding the relevent post. You have to go back a few times since the blog is updated regularly but it shouldn't take long, it was on page 5 when I looked for it just now (and you can always modify the page number in the URL to skip a few pages). Nil Einne (talk) 15:57, 24 October 2012 (UTC)[reply]
Thank you everyone - OP here. The fivethirtyeight link answers my questions perfectly. — Preceding unsigned comment added by Hisham1987 (talkcontribs) 16:13, 24 October 2012 (UTC)[reply]
(edit conflict)I know he said this was answered, but I just spent like 30 minutes working this out, so I'm posting it anyways, after the edit conflicts. AH! Sorry, in that case, assuming no state ends up going for a third party candidate, there are quite a number of scenarios leading to a 269-269 split. In general, most of the states aren't "in play", that is some states will vote Republican, and others Democrat regardless of who is running as a candidate (back in the days of the Solid South, this was referred to as Yellow Dog Democrats: people who would vote for a Yellow dog than vote for a Republican. This concept still applies to both parties in certain states, such that there is no realistic chance of having a close race in them). Looking at the article Red states and blue states and Electoral College (United States) we see the following breakdown, based on recent past performances of the parties:
  • Romney is probably a "lock" in the following states: Idaho 4, Montana 3, Utah 6, Wyoming 3, North Dakota 3, South Dakota 3, Nebraska 5, Kansas 6, Oklahoma 7, Texas 38, Alaska 3, Alabama 9, Mississppi 6, Georgia 16, South Carolina 9, for a total of 121 votes
  • Obama is probably going to win all of the following states: Washington 12, Oregon 7, California 55, Hawaii 4, Minnesota 10, Wisconsin 10, Illinois 20, Michigan 16, Maryland 10, Delaware 3, Pennsylvania 20, New Jersey 14, New York 29, Vermont 3, Massachusetts 11, Connecticut 7, Rhode Island 4, Maine 4, D.C. 3, for a total of 242
  • The following states are thus "in play", or "swing states" as they are popularly known: Nevada 6, Arizona 11, New Mexico 5, Colorado 9, Iowa 6, Missouri 10, Arkansas 6, Louisiana 8, Indiana 11, Ohio 18, Kentucky 8, Tennessee 11, West Virginia 5, Virginia 13, North Carolina 15, New Hampshire 4, Florida 29, for a total of 175
Several of the above "swing states" are probably more in one camp than the other, but this is probably the largest list of states where the election could go either way (realistically, I've seen several news reports that narrows it down to just 4-6 states: Ohio, Virginia, Florida, and North Carolina being the most important as they are closest to a 50-50 split, and have enough electoral college votes to make it worth the while. Also, there are a few "solid" states I wouldn't be surprised to swing this election. Georgia has a strong, affluent African-American population in and around Atlanta that could swing the state Democrat, likewise a state like Pennsylvania could also be in play for the Republicans, likewise Romney's home state of Michigan, or Massachusetts where he was Governor). However, just on the lists above, which gives the widest, most realistic chance of making the election, if every candidate took exactly the states they are supposed to, you could have the following swing states break the following ways:
  • Obama gets: Virginia (13), New Hampshire (4) Missouri (10) = 27 more than his "locks" = 269 total
  • Romney gets all the rest for 269 total.
There's your tie. There's probably many other ways it could break down for a tie, (especially if either Nebraska or Maine split their vote, as they do from time to time) but there's one realistic scenario. --Jayron32 16:24, 24 October 2012 (UTC)[reply]

Finding ties in the electoral college is an interesting example of a subset sum problem. Duoduoduo (talk) 16:25, 25 October 2012 (UTC)[reply]

For any given value of 'interesting'. --Jayron32 18:58, 25 October 2012 (UTC)[reply]

Commanders who never lost a battle

From the introduction of the Jan Žižka article: "Žižka is ... one of six commanders in history who never lost a battle (alongside Alexander the Great, Scipio Africanus, Genghis Khan, Alexander Suvorov, and Khalid ibn al-Walid)." The introduction of Alexander Suvorov lists Lucius Cornelius Sulla instead of Scipio Africanus. But other articles attribute the same virtue also to other military leaders, in a trice namely Paul von Lettow-Vorbeck and possibly Uesugi Kenshin. I assume the lists above to be original research and POV; where do they come from? Are there lists that might possibly claim justifiably to be complete? But then, there might be several military leaders who just won a single battle in their life and then died of a disease... any help to clear this is welcome. --KnightMove (talk) 08:37, 24 October 2012 (UTC)[reply]

This seems a rather vague thing to try to define. Would a commander who never fought a battle count ? How about a commander who only fought one ? Or a commander who avoided all battles except the easy victories ? None of these sound particularly heroic, do they ? StuRat (talk) 08:57, 24 October 2012 (UTC)[reply]
Yes, it's a pointless categorisation. Someone who only became involved in a war in its dying stages could easily score that label. HiLo48 (talk) 09:37, 24 October 2012 (UTC)[reply]
To say nothing of the fact that a number of the commanders above probably did loss a battle. Alexander the Great for example lost a number of engagements, it has just become common not to count any of them as a full battle, but in that case what are we really recording? 86.166.191.232 (talk) 09:41, 24 October 2012 (UTC)[reply]
I agree it is a pointless characterisation, and furthermore, unless it is directly stated in a reliable source, it is synthesis and should be removed from Wikipedia on that ground. I suggest you remove it; or discuss it on the article's talk page if you are not confident in doing that. --ColinFine (talk) 10:11, 24 October 2012 (UTC)[reply]
What about American commanders who led only in the Spanish-American War, say, and retired before WWI?--Wehwalt (talk) 12:14, 24 October 2012 (UTC)[reply]
It can't possibly be supported by a viable source in any event. Definitely a contentious claim and almost assuredly someone's original research; definetly needs to be removed. Snow (talk) 05:17, 25 October 2012 (UTC)[reply]
The list is blatantly incomplete. Marlborough is famous for never having fought a battle he didn't win, and never having besieged a city he didn't take. Looie496 (talk) 14:47, 24 October 2012 (UTC)[reply]
I've just removed this claim from the article per the above discussion. Nick-D (talk) 22:45, 25 October 2012 (UTC)[reply]
... and I did the same for Alexander Suvorov. --KnightMove (talk) 06:18, 27 October 2012 (UTC)[reply]

Why hasn't the electoral college system used in the United States caught on in the rest of the world?

With the US presidential election happening in a few weeks, this question came into my mind. The United States is extremely unique among presidential republics in that people don't actually directly vote for the president, but rather the people vote for a group of people who will make the decision for them. As I pointed out in another related question here, this is, to my knowledge, the only system of kind in the world (the electoral colleges of India and Pakistan being composed of lawmakers rather than a group of people elected specifically to elect the President). The closest equivalents would probably be the Election Committee of Hong Kong which elects the Chief Executive (although I'm not sure if the Committee's members themselves are elected or are chosen by interest groups) or the Assembly of Experts which elects Iran's Supreme Leader (although he isn't a president). Interestingly, the American sytle of government has been copied around the world, especially in South America, except for the electoral college. The question is, why? Why has the concept of an Electoral College not caught on in the rest of the world? I know that the College was a product of the Great Compromise, but if other countries can copy other elements of the US government (like having an executive president with veto powers), then why didn't they copy the Electoral College? Narutolovehinata5 tccsdnew 12:45, 24 October 2012 (UTC)[reply]

Why would they? What would be the advantages of the elctoral college system? The US is one of the oldest electoral democracies around, and as a result it has a highly anachronistic electoral system. --Soman (talk) 12:49, 24 October 2012 (UTC)[reply]
Honestly, I'm not a big fan of the electoral college system (I prefer direct elections), but I can see why they used it. it would benefit states with small populations. This would mean that even the electoral votes of, say Wyoming or Alaska, would have the same weight as the votes of Texas. Not really, but it would mean a more proportionate voting. That is why they had the Great Compromise in the first place. Narutolovehinata5 tccsdnew 12:56, 24 October 2012 (UTC)[reply]
(ec) It's not at all clear on the surface that the Great Compromise -- the disproportionate power granted to small states in the Senate -- is tied to the Electoral College. Without knowing the Federalist Papers offhand, I rather suspect that the reasons for the College shape closely to Article 1 Section 3, that is, the indirect election of Senators (which has no possible ties to state size). Note further that there is no constitutional requirement that Electors vote in accordance with, well, anything. Strictly speaking, there's not even a requirement that a popular election be involved in Elector selection (and this is a key component of the National Popular Vote Interstate Compact, which is attempting to tie state electoral votes to the winner of the national popular vote, regardless of state-level results). So no, it's not a Great Compromise thing, but rather a means of insulating the more powerful offices (i.e. anything but Representatives) from the common voter. I'm sure the Federalist Papers go into detail somewhere about why that gap was written in, both for the President and for Senators. — Lomn 13:48, 24 October 2012 (UTC)[reply]
Relevent Federalist Papers to the topic worth reading (a great insight to understanding the mindset and rationale of the Constitution writers themselves) are Federalist No. 49 and Federalist No. 51 (on the need for checks and balances generally), Federalist No. 10 (on the problem with factions, and on the need to institute measures in the government to combat factions, even factions of the majority, aka Tyranny of the majority), Federalist No. 39 is relevent, as a sizable portion of it deals with the inherent nature of whether the national government was intended to represent the Governments of the States, or the People directly: measures were built in to address both, but directly relevent is that the Presidency and the Senate were supposed to be representatives of the states, while the House of Representatives were representatives of the people. However, if we really want to get down to brass tacks, the relevent paper is Federalist No. 68, titled by Hamilton "The Mode of Electing the President", the text is availible at Wikisource if you want to read it, but especially germane here is the passage:

It was desirable, that the sense of the People should operate in the choice of the person to whom so important a trust was to be confided. This end will be answered by committing the right of making it, not to any preëstablished body, but to men chosen by the People for the special purpose, and at the particular conjuncture. It was equally desirable, that the immediate election should be made by men most capable of analyzing the qualities adapted to the station, and acting under circumstances favorable to deliberation, and to a judicious combination of all the reasons and inducements which were proper to govern their choice. A small number of persons, selected by their fellow-citizens from the general mass, will be most likely to possess the information and discernment requisite to such complicated investigations. (bold mine)

The founding fathers believed in representative democracy over direct democracy because they believed that the great masses of people were easily swayed by small, inconsequential things, and that it would be trivial to establish a majority which would decide issues that, in the end, would not be in the best interest of the country. The idea would be that wise, well-trained, and well informed individuals would themselves be selected by the people, and THOSE individuals would make all of the important decisions. The "electoral college" was intended to be as independent and vital as the Senate and House of Representatives; it was basically intended to be a third, special-purpose legislature which would meet, debate and deliberate, and then elect a president from their deliberations. That was the clear intent of the body from Federalist No. 68. Now, that it very quickly devolved into a mere formality is a historical fact, but was not something the Constitution writers envisioned or hoped for: the original intent was that the Electoral College would be chosen, not to represent specific candidates (as is done today), but to represent the people generally, and would through their collective wisdom and deliberation, elect a President themselves, but that the members of the College would be unencumbered by direct association with any party or faction. It is somewhat ironic that the actual operation of the College works exactly opposite of that: the members of the Electoral College are tied directly to the party they represent, and are in many cases actually forbidden by state law from voting independently. --Jayron32 14:30, 24 October 2012 (UTC)[reply]
Judging from what I read at the time of the Bush-Gore debacle, and from the article on the topic, the whole point was a compromise between secondary election (by individual states) and direct election (just one person - one vote). The downside is that it currently works much like a direct election, with this intermediate business of electoral colleges. I see no reason to imitate. IBE (talk) 13:40, 24 October 2012 (UTC)[reply]
(ec) And that (the fact that it was designed to suit the peculiarities of US politics back then) is in a nutshell why it has not caught on elsewhere.
The electoral college is mechanically similar to any other method of choosing a leader via electorates - for example the way the UK prime minister is chosen (voters voting in electorates for MPs, the candidate commanding the support of the largest number of MPs gets to be prime minister). However, in reality the electoral college is just a slightly less democratic version of a directly elected system. So in countries where there isn't the same precise pressues that led to the electoral college model being adopted (and, countries being as different as they are, these would be few and far between), it would be more expedient to simply choose a directly elected model, or an appointed model, or a parliamentary model, or a different kind of compromise to suit their own situation.
So, as the OP mentioned, the "electoral college" in Hong Kong is partly directly elected, partly elected by interest groups, and partly appointed. It is a different kind of compromise, this one between the pressues of democracy, interest groups and the Chinese government's wish for control over the process. --PalaceGuard008 (Talk) 13:43, 24 October 2012 (UTC)[reply]
I would also be cautious about simply regarding a country's system as "copied" from another. Most mature political systems are compromises of some sort, adopting elements of earlier systems and applying certain innovations or reforms. There are quirks to each country's system, so it would be rare to find one that's identical to another in any respect, not just the electoral college. --PalaceGuard008 (Talk) 13:47, 24 October 2012 (UTC)[reply]
Narutolovehinata5 -- the electoral college was well-adapted to the U.S. situation in the late 18th century and early 19th century: i.e. a semi-loose federation spread over a wide geographic area with relatively poor communications, where there was a legitimate concern that voters in one state might not know enough about politicians in other states to have an informed opinion on them, and there was little desire to have centralized government control to the degree that voting qualifications and election administration could be standardized between the individual states in a manner which would allow for throwing all the votes in the states together into one common national pool (a necessity for a national popular vote system). So as late as 1824 there weren't really national presidential candidates, but only a series of regional candidates; as late as the Dorr Rebellion of the early 1840s there was resistance in some areas even to Jacksonite "universal white manhood suffrage"; and as late as 1860, South Carolina didn't allow individual voters to vote for president at all.
Of course, there's no reason why what was found to be necessary for the U.S. in the late 18th century and early 19th century should be considered ideal for any other country in 2012... AnonMoos (talk) 14:04, 24 October 2012 (UTC)[reply]
That's fascinatng, AnonMoos -- as late as 1860, South Carolina didn't allow individual voters to vote for president at all. Do you mean just in 1860, or in all years up to and including 1860? Can you provide a reference about this? Thanks. Duoduoduo (talk) 14:38, 24 October 2012 (UTC)[reply]
It's also true: From 1788-1860 inclusive, South Carolina's state legislature selected it's Presidential Electors, without the use of a popular election. Start at United States presidential election, 1860 and work backwards and you can confirm that there was no popular vote in South Carolina. As you go backwards, you can also confirm that there was no popular vote in progressively more and more states. South Carolina was a pretty long hold-out in not using a popular election, but United States presidential election, 1828 shows that Delaware didn't hold a popular vote either, and United States presidential election, 1824 shows 6 states which appointed the electors in the Legislature, and United States presidential election, 1816 has a near even split: of the 19 states at the time, 10 used a form of popular vote, with the other 9 having electors appointed by the legislature. Prior to that, about half (more or less, depending on the number of states at the time) didn't have a popular vote. --Jayron32 15:24, 24 October 2012 (UTC)[reply]
After the 1824 election (whose outcome was considered deeply undemocratic by many, see Corrupt_Bargain#Election_of_1824), there was the rise of "Jacksonian democracy", which in presidential elections meant a close approximation to universal white manhood suffrage, and direct popular election of presidential electors -- Rhode Island and South Carolina being the main holdouts... AnonMoos (talk) 16:58, 24 October 2012 (UTC)[reply]
Please say "male sufferage". When you say "manhood sufferage" it makes it seem like only a specific part of the body is voting. Of course, that lends an interesting twist on the phrase "pulling the lever" (from the old "lever style" ballot-marking machines) to refer to voting. --Jayron32 17:37, 24 October 2012 (UTC)[reply]
And actually the word is "suffrage". I don't even want to think about "manhood sufferage"...--TammyMoet (talk) 17:57, 24 October 2012 (UTC)[reply]

I'd vote against that... --Jayron32 18:01, 24 October 2012 (UTC)[reply]
I didn't come up with the phrase -- see article Universal manhood sufferage... AnonMoos (talk) 19:28, 24 October 2012 (UTC)[reply]
Which redirects to the correctly spelt Universal manhood suffrage. Which in turn ought to be moved to Universal male suffrage because the "manhood" version of the terminology is outdated, but that's already a redirect to Universal suffrage. -- Jack of Oz [Talk] 20:53, 24 October 2012 (UTC)[reply]
"Universal manhood suffrage" was the historically-used term, and doesn't create any potential confusion about whether boys are allowed to vote... (By the way, I was spelling it correctly before I picked up on Jayron32's misspelling.) AnonMoos (talk) 00:23, 25 October 2012 (UTC)[reply]
In 2005 there was an unsuccessful proposal on Talk:Universal manhood suffrage to change manhood to male. Duoduoduo (talk) 16:47, 25 October 2012 (UTC)[reply]
I'm going to change the redirect for Universal male suffrage away from Universal suffrage to Universal manhood suffrage. Duoduoduo (talk) 16:47, 25 October 2012 (UTC)[reply]

Why? Because there doesn't exist a single country structured like America. America is unique in this respect, and is foremost a nation of states. During the Civil War, a lot of people chose sides based on what state they lived in, even if they disagreed with ideaology over slavery. Befor the American Revolution, people identified themselves not as Americans, but as a Virginian or a New Yorker... In orderto properly represent the views of all these states, rather than represent only the views of about 5 large states with large populations that can elect a president, the electoral college is a unique system that works. --Jethro B 00:02, 25 October 2012 (UTC)[reply]

Not exactly so. Switzerland was assembled much like America: originally as an association of essentially independent states that have gradually, over time, grown into a more federal (than confederal) relationship. See Old Swiss Confederacy for some perspective. South Africa also has its history as a voluntary federation of independent states, but it is today a unitary state. America is certainly an unusual type of nation, given that more states are "unitary states" without any subnational unit having true sovereignty, and most of the remaining federations have a "Strong federal" model whereby the National law supercedes State/Provincial law in more cases than not: In countries like Canada, unless something is specifically assigned to the Provinces as their responsibility, it is reserved to the Federal government. The U.S. has a "weak federal" model whereby the National government has only a series of "enumerated powers" and other than those specific powers and roles (such as military, currency, interstate commerce, etc.) all other sovereign powers are specifically reserved to the states. Unusual, but not sui generis. Other states with similar arrangements include the aforementioned Switzerland, the United Arab Emirates and Malaysia. --Jayron32 03:08, 25 October 2012 (UTC)[reply]
Every country is different. Australia is also an assemblage of formerly separate colonies, and federal powers are also enumerated in the constitution, either as exclusive to the federal government or as shared by the federal and state governments (but with federal laws prevailing to the extent of any inconsistentcy). All other powers not so enumerated is reserved to the states.
However, Australia has never had a need for an electoral college system because the Australian head of government is selected by the majority parliamentary party from among its ranks, and the Australian head of state in practice (the Governor-General) is mostly ceremonial and nowadays basically appointed by the government.
Every country is different, and no doubt reasons also exist why the other countries you listed would not have adopted an electoral college system. --PalaceGuard008 (Talk) 09:35, 25 October 2012 (UTC)[reply]
And also relevant — the Governor-General has never been an elected position; regardless of who chooses him (government or Colonial Office), he's always been appointed. Nyttend (talk) 15:08, 27 October 2012 (UTC)[reply]

female muezzin and female imams reading the prayer out loud during Maghrib, isha and fajr prayers.

What does islam say about women being muezzin for women-only congreagtion only and also, if a woman leads the maghrib, isha and fajr prayers for the women-only congregation, does she read the prayers out loud or not? — Preceding unsigned comment added by Donmust90 (talkcontribs) 16:20, 24 October 2012 (UTC)[reply]

I expect that someone who knows more about this than I do will be along shortly. But just as with the earlier question about female clergy in Christianity, there's no single unified position about this within Islam. I remember reading recently about a large programme in Turkey to train women for ministry as imams. This was met with a wide range of reactions, and this reflects the complexity and diversity of opinion - popular and scholarly - even in such a clearly Muslim-majority country as Turkey. AlexTiefling (talk) 16:32, 24 October 2012 (UTC)[reply]
Wikipedia articles Islamic_feminism#Equality_in_leading_prayer and Women as imams... -- AnonMoos (talk) 17:05, 24 October 2012 (UTC)[reply]

October 25

the price of David Copperfield's caul

What does "winner to spend" mean? That the winner receives 5s spending-money? Or that the winner, to claim the grand prize, must put up another 5s (twice the price of the raffle ticket)? —Tamfang (talk) 00:36, 25 October 2012 (UTC)[reply]

Sounds like he won an option. μηδείς (talk) 02:55, 25 October 2012 (UTC)[reply]
Or the bidders are submitting binding offers, with the winning bid selected by lottery. It is not clear just from the passage quoted whether the winner, once selected, has the option of backing out of the purchase or not (if they do, it would be an option, if not, it wouldn't be.) --PalaceGuard008 (Talk) 09:28, 25 October 2012 (UTC)[reply]
Speaking purely as a reader (I think I may have formally studied the novel at school, but that was a looong time ago), I understood it to mean your second alternative, that the winner would have to pay an additional 5/- over and above his 2/6 raffle ticket in order to claim the prize: thus the organisers would (before expenses) gain 50 x 2/6 + 5/- = £6 10/-. {The poster formerly known as 87.85.230.195} 84.21.143.150 (talk) 13:27, 25 October 2012 (UTC)[reply]
For any reader wondering what on earth this is all about, our Caul articles says that it is "...is a piece of membrane that can cover a newborn's head and face immediately after birth." It continues; "A legend developed suggesting that possession of a baby's caul would give its bearer good luck and protect that person from death by drowning." The winner of David Copperfield's caul was an old lady who "... was never drowned, but died triumphantly in bed, at ninety-two." Alansplodge (talk) 17:46, 25 October 2012 (UTC)[reply]
Thanks, I was wondering about that, and why anyone would want a magician's caul. :-) StuRat (talk) 18:28, 25 October 2012 (UTC)[reply]
[12] - for anyone who didn't understand StuRat's allusion. (Like me, for ex.) -- Jack of Oz [Talk] 23:11, 26 October 2012 (UTC) [reply]
Why not charge an extra penny and avoid the winner having to pay the five shillings?--Wehwalt (talk) 23:41, 26 October 2012 (UTC)[reply]
Not sure but a) there was a 2s 6d coin so it would have been easier to collect (and account for). b) The caul was originally offered for 15 Guineas ie £15 15s (£15.75p decimal) so even at 7s 6d (37.5p) it was still a considerable bargain. Alansplodge (talk) 22:46, 27 October 2012 (UTC)[reply]
Now that I think about it, I think lotteries required leave of the government or an act of Parliament or some such. If the prize is an option (which, of course, could be declined), it may avoid that problem.--Wehwalt (talk) 22:50, 27 October 2012 (UTC)[reply]

Roman Catholic incorporation?

In The Castilian Fathers at the Council of Basel, it states "Cardinal Cervantes incorporated on November 29, 1432 along with three members of his household." Obviously, he didn't go into business. What does this mean? Clarityfiend (talk) 05:30, 25 October 2012 (UTC)[reply]

I think what we're talking about here is College (canon law). Marnanel (talk) 05:43, 25 October 2012 (UTC)[reply]
Makes sense. Thanks. Clarityfiend (talk) 06:52, 25 October 2012 (UTC)[reply]
Resolved

The mechanics of the Imperial Diet of the HRE

So, I was just reading through some articles about the Holy Roman Empire, and while perusing through List of Reichstag participants (1792), I had some questions about how the Reichstag worked during this time period. Reading Imperial Diet (Holy Roman Empire) doesn't give a lot of insight as to the mechanics of how various Diets operated, excepting to note that "The precise role and function of the Imperial Diet changed over the centuries" without noting the operation of the Body. In List of Reichstag participants (1792) there are several things I have questions about regarding how that Diet would have operated, based on the voting membership:

  • Several "seats" in the Diet are held by the same person. For example, Archduke Maximilian Francis of Austria has several seats on the body: One as Elector of Cologne, a separate one as Bishop of Munster, and another as Grand Master of the Teutonic Knights. When Maximilian Francis officially recorded a vote, did it represent the vote of one man (as his vote) or the did it count as 3 votes (one for each office he held)?
  • Several members of the Diet are foreign princes who held important offices elsewhere. For example, many of the seats are held by George III of the United Kingdom through his various German titles. However, George never set foot in Hanover, and I don't believe he attended the Diet in person, which brings up the question of his votes: Could he "vote by mail", sending envoy of his official vote on various issues, or could he "vote by proxy" by sending a representative in his stead to represent his voice, and vote for him? And if so, did he send one envoy (to represent him) or several (to represent each state he had title to)? Or, if he wasn't personally present, did he forfeit his right to vote?
  • Some seats are held simultaneously by more than one person, sometimes many people combined for one vote. For example, the vote held by The Duke of Saxe-Coburg was officially split between two men: did they have to achieve consensus to register their single vote, or did each man represent 1/2 vote in the tally? The vote of The Prince of Anhalt is similarly divided among 4 people, and others similarly.
  • Regarding the "Colleges" Were they handled differently for the split votes of some Duchies noted above, which collectively represented a single vote? I suspect, from the wording of our articles, that the single vote of the College of The Counts of Swabia would be arrived at by the College voting amongst itself first, and then recording that as their one Vote in the diet? Or was there a representative elected from the college that served as the person who made the votes in the diet? Or did one of the members of the College have hereditary right to represent it in the diet?

Just some questions that come to mind. Any ideas or thoughts? --Jayron32 18:46, 25 October 2012 (UTC)[reply]

Erzherzog Maximilian Franz von Österreich didn't inherit any votes, he was a self-made Diet seat-holder. All his votes weren't hereditary. I'm guessing the votes were cast in order of precedence, the name of the seat was called out and the holder stood and cast his vote for all to hear. I'm guessing if the King of the United Kingdom of Great Britain and Ireland didn't send a deputy then he would lose his multple votes. I'm guessing the split votes would require agreement before the vote or the holders would alternate each vote. For the circles that held one vote, I'm guessing the majority of an internal vote was used or maybe rounds like the election of the Pope.
Sleigh (talk) 21:21, 25 October 2012 (UTC)[reply]
There is a category de:Kategorie:Funktionsweise Reichstag (HRR) with a couple of small articles or stubs on the functioning of the diet, the corporate votes were called Kuriatstimmen and the personal votes Virilstimmen. --Pp.paul.4 (talk) 22:01, 25 October 2012 (UTC)[reply]
Can you translate, perchance, the relevent bits? I don't have much German skills. Danke schoen. --Jayron32 22:09, 25 October 2012 (UTC)[reply]
My best guess is: (Question 1) Counted as 3 votes. (Question 2) I do not think that personal presence ever played a role. Everybody was represented. (Question 3) By consensus. (Question 4) Internal consensus reached first. Please correct. --Pp.paul.4 (talk) 22:55, 25 October 2012 (UTC)[reply]
What am I supposed to correct? Do you have any documents or sources which inform your guesses? --Jayron32 01:54, 26 October 2012 (UTC)[reply]
Everybody is supposed to correct my answers, for I have only marginal knowledge in this realm. The answers are confirmed, however, by Reichstag in de:Universal-Lexikon der Gegenwart und Vergangenheit, online [13] or [14]. --Pp.paul.4 (talk) 12:28, 26 October 2012 (UTC)[reply]
Thanks! Your answers are helpful then! Just one question for clarification. Your answer for number 2 is unclear. When you say " I do not think that personal presence ever played a role. Everybody was represented." Does that mean that "Every person who had a seat in the Reichstag was represented by somebody other than themselves" or does it mean that "Every person who had a seat in the Reichstag was represented, some in person and others by proxy"? If the second, was the "proxy" a single person per seat or per representative: That is, was George III represented in the Reichstag by a single proxy to represent him, or by one proxy for each of his German realms? --Jayron32 13:33, 26 October 2012 (UTC)[reply]
If we do not talk about the periodical diets of 1486-1654, but about the Perpetual Diet of Regensburg 1663-1806, I guess the Emperor never attended this diet in person, hence the electors and princes saw no need to attend it in person, hence everybody sent representatives in the diplomatic rank of envoys (not even ambassadors). Such an envoy (I would not compare this to proxy voting) represented one or several votes, even of different princes, see also PDF with images. --Pp.paul.4 (talk) 13:13, 28 October 2012 (UTC)[reply]
(outdent) - If this was anything like the Imperial electoral college, there would be a rigorous process for the acknowledgement of proxy voters, and each prince would only need one proxy, no matter how many votes he held. (I suspect that the split and collegiate votes would result in the delegation of one proxy voter each, too - but I have less basis for this.) And remember: in the United States, it's your vote that counts. In the HRE, it's your Count that votes. AlexTiefling (talk) 14:41, 26 October 2012 (UTC)[reply]

Old people over here

I just want to know whether there are old people on here. I'm 77 and I remember it clearly to hear on the radio about the death of last surviving veterans, such as Albert Woolson of the American Civil War, or Hugh Theodore Pinhey, of the Second Anglo-Afghan War, or other people's death such as that of Samuel J. Seymour, who was the last witness to the assassination of President Lincoln. I remember it all. Is there anyone on here with the same memories? Thank you. Iowafromiowa (talk) 19:12, 25 October 2012 (UTC)[reply]

I'm sure there are other people your age on here. For my part, I'm acutely aware that (if I'm spared) I will one day be in your position. As it is, I'm old enough to be slightly awed by the people here who don't remember the Cold War, Thatcher and Reagan, and so on. AlexTiefling (talk) 20:44, 25 October 2012 (UTC)[reply]
Then you'll be really awed by people like me, who don't remember 9/11. --140.180.252.244 (talk) 10:30, 26 October 2012 (UTC)[reply]
It's chilling to realise the Beatles formed half a century ago, and last performed 43 years ago. And Lennon's been dead for almost 32 years.
I remember going to Anzac Day marches where veterans from the Boer Wars marched, but I don't remember hearing when the last Boer veteran died. They didn't seem to make as big a thing of such "last events" back then as they do now.
I was born in the dying months of 1950. The first historical event I remember was the arrival of television in Australia in September 1956. We lived in the country and didn't have TV till about 1963, but all our Sydney relatives got them early, and it was always a big talking point when the next lot of rellos got a TV. Ironically, I have no memory of the 1956 Melbourne Olympics, which was why TV was necessary, but it would have been radio news in our case, rather than TV broadcasts, so .... I imagine the first time I actually saw television was at Christmas 1956 when we visited Sydney, but the Olympics were over by then. I clearly remember hearing about the launch of Sputnik 1 in October 1957. I remember often hearing on the radio about some dude named what sounded to my tender ears as "President Ivanhoe". I have a vague memory of hearing when Oliver Hardy died in 1957, and when Mike Todd was killed in March 1958 (but mainly because he was Elizabeth Taylor's husband - yes, she was very well known even to 7 year old boys). But the first clear memory I have of a notable person dying was that of Pope Pius XII in October 1958. Church bells rang unexpectedly, and we didn't have to come to school the next day. Yippee. -- Jack of Oz [Talk] 20:53, 25 October 2012 (UTC)[reply]
I was born in 1963. The first historical event I remember knowing about was Expo '70 in Japan, I wanted my parents to take me there from New Jersey ... when I visited Osaka in 2002, I was careful to visit the site. The first death of a notable person I remember was J. Edgar Hoover, I was home from school (1972) and it was on TV. I remember watching a moon landing on TV, not certain which one, but my late mother told me that I fell asleep for Armstrong's small step. I suppose I became aware more of the world in 1972, as I remember watching Avery Brundage's famous Munich speech on September 6, 1972 (perhaps it was on tape). Also, my father had a reel-to-reel audio and 8-track tapes, neither of which he used often.--Wehwalt (talk) 21:08, 25 October 2012 (UTC)[reply]
Though I prefer not to give my precise age, like JackofOz I'm a sexagenarian and pretty active here (though not nearly so much as Jack, sheesh). There's not anywhere nearly so many of us as there are the young whippersnappers, but we're represented (and a lot wiser, heh, heh, heh). I don't have the memories you do, you've got a few years on me, but I can remember some things just short of those. (43 years since the last Beatles performance? I hadn't realized that until now. Now I'm depressed...) Best regards, TransporterMan (TALK) 21:17, 25 October 2012 (UTC)[reply]
I listened live to the radio broadcasts of Marilyn Bell's swim across Lake Ontario. Bielle (talk) 21:22, 25 October 2012 (UTC)[reply]
I remember where I was when I heard about the Assassination of John F. Kennedy - I was 5 years old. I also remember watching the funeral of Winston Churchill on TV, I had reached the grand old age of 6 by then. Alansplodge (talk) 14:39, 26 October 2012 (UTC)[reply]
Seriously, you guys make me depressed to think that one day, I'll be as old as you. I'm not old enough to remember any major historical events except the 2008 financial crisis; my earliest memory was right after 9/11, which I wouldn't even call a major event. The Cold War might as well be the Peloponnesian War from 2400 years ago, because that's how much I can relate to either (and actually, if you read Thucydides, it's depressing how similar the Athenian/Spartan justifications for war mirror those offered for the "hot" proxy wars by the US/USSR). Considering the murderous passions that historical conflicts seem to incite in people generations after they take place, I'm very glad that I don't remember any. --140.180.252.244 (talk) 21:29, 25 October 2012 (UTC)[reply]
Even more depressing is to think that in 20 years or so, your "nostalgia" radio channel will be playing such eternal classics as "Call Me Maybe" and "Gangnam Style". ←Baseball Bugs What's up, Doc? carrots02:37, 26 October 2012 (UTC)[reply]
Hey, those will be widely praised after their creators die, just like a lot of famous works. Also, radio channels? If every square centimeter of my beloved city doesn't have high-speed WiFi coverage by 2022, I'll make it happen. --140.180.252.244 (talk) 10:30, 26 October 2012 (UTC)[reply]
Funny, I just had a nightmare about a nuclear war two nights ago. That dates you. My earliest historical memories are of the last moon shot and Watergate. Elvis's death and Jonestown I remember clearly. John David Stutts shooting Reagan and Mehmet Ali Agca shooting the Pope happened a few months ago, and 9/11 last week. μηδείς (talk) 21:52, 25 October 2012 (UTC)[reply]
As you grow older, hopefully you'll come to realize it's even more depressing not to reach our Mathusalean age. Clarityfiend (talk) 22:22, 25 October 2012 (UTC)[reply]
After you have lived about maybe 80 years to 100 years then you will have a feeling of you have lived enough. Seriously, I wouldn't live pass 200 years, it is just too much. Death always has been the natural cycle of life, you can't escape it no matter what. Even if one day we, humans, somehow can make ourselves live forever then that would defy natural and to me it is not a good thing. And I'm very sure when you live to a certain point you WILL want to die. Even the most lively people will want to die "eventually". Maybe not after 1 thousand years or 1 million years or 1 billions or 1 trillions.... Eventually you will want to die, you got my point. Perhaps that was the reason Albert Einstein, a genius, who has well understood the meaning of life, refused to get a surgery to prolong his life (doesn't matter how long the surgery could have prolonged his live but the point here is he doesn't want to prolong his life even for 1 second). When it is time to go, just be happy to go, that's my way of taking it. Just live a meaningful life so when you died you will feel the fulfillment of life! That's all there is to the world! 184.97.240.247 (talk) 03:48, 26 October 2012 (UTC)[reply]
Malthusian? Methuselahian? -- Jack of Oz [Talk] 22:35, 25 October 2012 (UTC)[reply]
As in like Mathusal, Methuselah's older, illiterate brother. Clarityfiend (talk) 22:57, 25 October 2012 (UTC)[reply]
I remember watching The Day After as a college student (1983), everyone in the dorm got together and watched it in the lounge. Very somber, as I recall, lots of crying, even a few of the guys.--Wehwalt (talk) 00:48, 26 October 2012 (UTC)[reply]
I just watched that again over the summer, great film. And it did influence my nightmare. Probably the best work the KGB ever did. μηδείς (talk) 02:02, 26 October 2012 (UTC)[reply]
I think that would surprise the writer, who is (or was) a card-carrying Republican. Oh, I forgot: anything to the left of Genghis Khan is commie. --NellieBlyMobile (talk) 04:15, 26 October 2012 (UTC)[reply]
I am not a Republican, your comment on party affiliation is silly. It's documented that the KGB led a misinformation campaign aimed at getting the American left to support unilateral disarmament. Carl Sagan's politically premissed nuclear winter paper data was never published. According to Sergei Tretyakov,[15] the KGB held a party to celebrate the showing of The Day After, which was coordinated with the nuclear winter propaganda campaign and a special on ABC which ran the miniseries. As for political labels, I am not interested. If you think there's some sort of essential distinction between the murderous dictator Genghis Khan and the murderous dictator Joseph Stalin your thought is too subtle for me. Nevertheless, The Day After was a great movie. μηδείς (talk) 05:52, 26 October 2012 (UTC)[reply]
IMHO, Genghis Khan was no more a dictator than he was a misogynist or a homophobe Asmrulz (talk) 14:25, 26 October 2012 (UTC)[reply]
When I was young the world population was just over a third of what it is now. I lived in a jungle and spoke another language. Now the jungle is cut down and there is a town there. My father and mother went back there a few years ago and found a couple of pictures of themselves hung up in the town hall, the people were astonished some of the 'ancestors' were still alive! Dmcq (talk) 10:19, 26 October 2012 (UTC)[reply]

There are always some things that seem to define a generation, that younger people on hearing it think you are really old. When I was a kid (born 1960) it was people who remembered WW2. In my case people even a decade younger than me are amazed that I can remember steam trains on British rail. My daughter says that people a few years younger are amazed when she talks about "before there were mobile phones" or the internet. Somehow to each generation each thing feels like it is in the distant past, even though it was not that long ago. -- Q Chris (talk) 14:38, 26 October 2012 (UTC)[reply]

The phenomenon of remembering significant and often unexpected events in vivid detail is called flashbulb memory. For what it's worth, I was born in Australia in 1987, and the first event I remember vividly was relatively local in nature – the rescue of Stuart Diver after the 1997 Thredbo landslide; . I also vividly remember the slightly later death of Princess Diana and being bitterly disappointed that the TV show Who Dares Wins was cancelled due to coverage of her demise. However I only have vague memories of the Port Arthur massacre and, going back a bit further, I can't remember much at all about the Oklahoma City bombing or the Srebrenica massacre. I'm totally blind as it says on my user page, so this doubtless affects my perception and memory of TV news reports. Graham87 15:19, 26 October 2012 (UTC)[reply]

I can also vaguely remember former Australian Prime Minister John Howard's acceptance speech after his party won the 1996 election; some of my classmates in high school seemed pretty amazed at that. But all I can remember him saying is that he'd support people with disabilities ... enough blathering from me. :-) Graham87 15:34, 26 October 2012 (UTC)[reply]
Interestingly, the article Childhood amnesia says: "Research has suggested that public events are remembered from approximately age 6". Graham87 16:58, 26 October 2012 (UTC)[reply]

Why does Obama vote in Chicago?

Why not Washington, where he lives, or Hawaii, where he was born? OsmanRF34 (talk) 21:27, 25 October 2012 (UTC)[reply]

If he does, that is because Chicago is where he is registered. Bielle (talk) 21:29, 25 October 2012 (UTC)[reply]
The basic answer is that when a person has a temporary residence, like Obama does in DC or while you're in college or on an extended but temporary job assignment, the law in the US affords quite a bit of choice to you to decide what your permanent residence (i.e. your domicile) is going to be. That determines where you vote. Ordinarily, and most easily, you'll either choose between where you are now, with the idea that though it started out as being temporary you don't intend to leave when the temporary situation ends) or the last place you did live as a permanent residence. That, in Obama's case, was Chicago. Regards, TransporterMan (TALK) 21:38, 25 October 2012 (UTC)[reply]
"Choice" — in practice, maybe. In theory, no. States have definite rules about whether you are a resident or not, and choice does not enter into them. "Intent", on the other hand, does enter into them, but your intent is a question of fact, and in theory they could attempt to prove that you are a resident even though you say you're not (or vice versa). --Trovatore (talk) 21:55, 25 October 2012 (UTC)[reply]
College students frequently vote in the district of their permanent residence and not where they spend the night when on campus. Living in a dormitory on a college campus doesn't make a student automatically and unambiguously a resident of the district where the dorm is located, and not a resident of, say, their parent's home where they used to sleep most nights. Presidents traditionally always vote in their "official" home district, and not Washington DC. A quick perusal will easily show George W. Bush voting in Texas, Bill Clinton voting in Arkansas, etc, etc, back through time. Obama isn't doing anything that every president prior to him didn't do.--Jayron32 22:07, 25 October 2012 (UTC)[reply]
On the other hand, the courts have pretty regularly upheld that students are allowed to vote in their college precincts, even if they live in, for example, a dormitory [16]. It really is, in most cases up to the student what they consider to be their residence. Buddy431 (talk) 23:16, 25 October 2012 (UTC)[reply]
Your link doesn't indicate, at least literally, that a student can choose where he/she votes. It says it depends on whether the student intends to return to the parental home on graduation. Of course, in many cases that's not particularly well-defined, but in theory a student who clearly intends to return home after graduation is supposed to vote there rather than where the school is. And conversely, a student who definitely does not intend to return home is probably not supposed to vote there. --Trovatore (talk) 00:37, 26 October 2012 (UTC)[reply]
True. My hometown being in Salina, Kansas that's where I cast my ballot (via absentee ballot) even though I have the choice to register as a voter in Norman, Oklahoma. I've even seen voter registration drives on campus the past couple years. I feel like I have more of a say casting my ballot back home than I do at university, so I stay registered at home. Ks0stm (TCGE) 00:28, 26 October 2012 (UTC)[reply]
There's not much about it in our Wikipedia article, but in Texas Waller County is semi-notorious for repeatedly down the years coming up with semi-ingenious schemes to inconvenience students at a historically-black college in the county from voting there (a right which they indisputably have under relevant state laws)... AnonMoos (talk) 22:56, 25 October 2012 (UTC)[reply]
No one cares where they vote for president. The issue is voting for local officials by out-of-towners living off and with their parents. That's a concern in every college town I have lived in. μηδείς (talk) 23:11, 25 October 2012 (UTC)[reply]
"Out of towners" who live there the majority of the year? The courts have consistently upheld the rights of college students to vote in the elections of their school's district if that's where they consider themselves to be residents. Buddy431 (talk) 23:19, 25 October 2012 (UTC)[reply]
I am not arguing for or against, just pointing out that the concern does exist, since yes, the interests of temporary student residents and families and property owners can certainly clash. μηδείς (talk) 01:48, 26 October 2012 (UTC)[reply]
It becomes an issue when the kids have their parents' state's drivers license, which is a privilege of state citizenship (or, in a few cases, employment). That's one of the issues over voter ID laws.--Wehwalt (talk) 23:25, 25 October 2012 (UTC)[reply]
A driver's license is not a prerequisite for voting. I'll be voting this year in the state that I go to school, while my driver's license is from the state where I used to live. An ID at the polls (what most current voter ID laws deal with) is to prove that you are who you say you are, not that you're a resident of the precinct (they check that when you register to vote). Buddy431 (talk) 23:31, 25 October 2012 (UTC)[reply]
You might want to look into what your new state has by way of requiring that you get a local license when you move into a new state ... I think there's something about it in the interstate compact on drivers, which is probably part of your state's law ...--Wehwalt (talk) 23:42, 25 October 2012 (UTC)[reply]
Sure, my state has rules about how quickly I have to get a new driver's license (90 days). But that has nothing to do with whether or not I am eligible to vote. Buddy431 (talk) 00:04, 26 October 2012 (UTC)[reply]
Both are generally privileges of state citizenship, though different time periods for a change on moving are involved, and it is possible to get a driver's license from a state sometimes though you are not a resident.--Wehwalt (talk) 00:52, 26 October 2012 (UTC)[reply]
It's probably got more attention this time because he voted early, which I think is a first for a president. When the spotlight shines, it illuminates everything. -- Jack of Oz [Talk] 22:33, 25 October 2012 (UTC)[reply]
So, did he vote Green, or Socialist? μηδείς (talk) 23:08, 25 October 2012 (UTC)[reply]
straight Official Monster Raving Loony Party ticket. Blueboar (talk) 00:17, 26 October 2012 (UTC)[reply]

(removed, just saw the immediate preceding post) The Masked Booby (talk) 00:33, 26 October 2012 (UTC)[reply]

I hear he wanted to write in for vice president and they wouldn't let him.--Wehwalt (talk) 00:54, 26 October 2012 (UTC)[reply]
I heard... well actually, I didn't hear, because he voted by Secret ballot. He can write in whoever he wants, and they'll only determine the validity of the ballot and the vote anonymously, when all votes are tallied. Buddy431 (talk) 01:24, 26 October 2012 (UTC)[reply]
You can't actually write in for vice president as you are voting for electors, so it was meant humorously.--Wehwalt (talk) 02:03, 26 October 2012 (UTC)[reply]
I can write in for vice president on my Kansas ballot...if you chose to write in there are blank lines for you to write in both the president and vice president you're voting for. Ks0stm (TCGE) 02:19, 26 October 2012 (UTC)[reply]
Interesting. Can you vote Obama for pres but someone else besides Biden for vp?--Wehwalt (talk) 02:27, 26 October 2012 (UTC)[reply]
In my state, there are several paired choices, and then a single line for a write-in. Obviously, write-ins have no chance of winning the popular vote. But if they did, the state would have to somehow come up with a slate of electors. ←Baseball Bugs What's up, Doc? carrots02:35, 26 October 2012 (UTC)[reply]
I saw an odd possibility in the paper. If there is an exact 50-50 split in the electoral college, the election is decided by the two houses of Congress, which will presumably vote on party lines. The House of Representives elects the president (1 vote for each state, presumably decided by the party that controls that state's delegation) and the Senate elects the president. Anyway the numbers work out that the House is Republican controlled and would pick Romney, while the Senate is Democrat controlled and would pick Biden. So we'd have Romney as President and Biden as VP. What a weird thought. 67.119.3.105 (talk) 07:30, 26 October 2012 (UTC)[reply]
It's the new House and the new Senate, so that's a bit more up for grabs than you note. Running quickly down the list of current representatives, I get 31 states with a majority of Republicans in the House, 16 states Democratic, three deadlocked (though one deadlock, Washington, is created by a vacancy caused by the death of a Democrat, so if form holds, that state would go Obama in a House vote.--Wehwalt (talk) 12:44, 26 October 2012 (UTC)[reply]

(Outdent.) Something has just occurred to me: suppose you lived (say) in both Philly and Chicago, because you were a college student. Would it be possible to have a vote in electing the mayors of both cities? If you would, is there any theoretical or practical reason you could not also have a vote in electing the presidential electors from both PA and IL? Marnanel (talk) 04:00, 26 October 2012 (UTC)[reply]

Is Chicago really known for its stringent election rules? --NellieBlyMobile (talk) 04:08, 26 October 2012 (UTC)[reply]
Well... --Jayron32 05:26, 26 October 2012 (UTC)[reply]
I see dead people ... voting. Clarityfiend (talk) 01:47, 27 October 2012 (UTC)[reply]
Off topic, but it might be interesting to people to know that in the UK, you can register to vote at more than one address if you spend "an equal amount of time at each". This is frequently done by students. In a general election, you can choose which of them to vote in (and you don't have to decide in advance, just turn up to one or other polling station on the day). In local elections, you can legally vote in both locations on the same day, if you so choose. --OpenToppedBus - Talk to the driver 10:20, 26 October 2012 (UTC)[reply]
Marnanel -- at one point I was registered in two different counties at the same time (though I didn't want to be). If I had voted in both counties for the same election, then I would have committed a crime... AnonMoos (talk) 10:36, 26 October 2012 (UTC)[reply]

October 26

I know this company was broken into 33 different companies. I wonder how the process happened. Like who are the 33 different people that each own a company? Did John D. Rockefeller choose those people? Can those 33 people are all Rockefeller's descendants? And if his company was broken into 33 different companies so is that mean he is going to lose 32 out of 33 amount of his fortune? 184.97.240.247 (talk) 02:08, 26 October 2012 (UTC)[reply]

Standard Oil wasn't owned by one person. It was a Joint-stock company which means it was owned by its shareholders. Anyone that owns stock in a company is an "owner". Rockefeller didn't own Standard Oil, as one might "own" a car. He owned a lot of shares of Standard Oil, was a founder of it, and served as its President and CEO, but founding a company is not the same thing as owning a Sole proprietorship. The specific ownership structure of Standard Oil in the early days is described at Standard Oil. Being the founder of a company is more like being the founder of a club: there are many stakeholders and many owners (those people who owned shares). Rockefeller owned enough shares to have a Controlling interest in the company. In his position as President and CEO, he directed the company through its major business decisions, but again it wasn't "his company" in the sense that he owned it as one owns an object. Standard Oil explains his role pretty well in the opening. It is also important to note that Standard Oil was basically a Holding company: it's main operation was the aquiring and integration of other companies into itself, or the founding of specific companies with specific purposes. So, Standard either bought an existing company, or would found a new company (wholly owned by Standard Oil) to cover some operation, either to market specific oil products in specific geographic locations, or to handle some aspect of the exploration, transportation, or processing of crude oil. Thus, the "break up" of Standard Oil wasn't arbitrary; the new companies already operated as semi-autonomous units within the Standard Oil Trust. What happened was that these companies were legally seperated into truly autonomous units, which were supposed to run seperately so they had to compete with each other and with other, non-Standard oil companies. The way this worked was that individual "Baby Standards" were granted regional monopolies over the use of the "Standard" name in a certain geographic area, but that's pretty much how Standard worked pre-break up. The only thing that really changed via the breakup was the corporate governance and ownership structure: The companies sold their own shares and had their own independent governance, and there was no industry-wide national coordination between them, though many of the companies did coordinate bilaterally (for example only a few of the Baby Standards were actual suppliers as well as marketers: Kyso stations sold Exxon and Mobil products which were often rebranded as "Kyso", but they were manufactured by Exxon or Mobil) this page isn't strictly a "reliable source" by Wikipedia standards, but it does have some good information on the history of the growth, domination, and later dissolution of the Standard Oil trust, as well as the modern fates of all of those companies. --Jayron32 02:54, 26 October 2012 (UTC)[reply]
Alright to my understanding of your explanation, let me sum it up. So before the breakup there were many small companies that were part of the Standard Oil and after the breakup they became independent companies. So do those small companies within the Standard Oil has to pay the Standard Oil to breakup? Did the Standard Oil gain any money from the breakup? Or did it actually lose trusts in the companies without being paid?184.97.240.247 (talk) 03:29, 26 October 2012 (UTC)[reply]
That's a good question. Since anti-trust court proceedings are civil and not criminal, it would seem that prima facie even without a direct link to answer your question, that the shareholders of Standard Oil could not have been deprived of their property without receiving equal value. So Rockefeller et al would have lost control of the Baby Standards from a corporate governance point of view, but they could not be deprived of the value of their shares when the breakup occured. As to the specific means of their compensation, I don't have any direct evidence yet (i'll look), but I suspect that there are a number of ways that could have happened. First, they could have been given fair market value of their shares; i.e. forceably "bought out" of their company by being provided cash at "fair market value" based on reasonable estimates of the value of the company: I.E. if Rockefeller had owned 30% of the company, he would have been given cash equal to %30 of the value of the company. The mechanics of how that cash would be acquired, and what would have happened to his shares (i.e. who would have bought them) sounds like a nightmare. The other possibility I can think of (one that seems more likely) is that he was given equivalent non-voting shares equivalent to his former position in the companies: He still owns the same value of shares (the same part of the companies) but as his shares are no longer voting shares, he has no say in corporate governance. I'll do some searching to see if I can find any harder sources on what actually happened. --Jayron32 04:55, 26 October 2012 (UTC)[reply]
This source from The Economist doesn't directly answer the question, but has an interesting quote:

On May 15th 1911, Rockefeller, again out on the golf course, was told that the Supreme Court had found the firm guilty of antitrust violations, and ordered it to be broken up. “Buy Standard Oil,” he advised his playing partner. A good tip: its pieces proved to be worth far more apart than together.

In other words, if my second scenario holds true, financially the windfall would have benefited Rockefeller greatly, as he stood to make a LOT of money on his shares in the new, separate companies, even if they were non-voting shares. Still looking for more. --Jayron32 04:59, 26 October 2012 (UTC)[reply]
This source of somewhat uncertain reliability, does state "The Court ordered the Standard Oil Trust to dismantle 33 of its most important affiliates and to distribute the stock to its own shareholders and not to a new trust." That is, the stock of the new companies was distributed to the existing shareholders of the old Trust. More evidence for my second explanation, though it doesn't indicate the nature of those shares (voting, nonvoting, common, preferred, etc.) --Jayron32 05:03, 26 October 2012 (UTC)[reply]
More: "Although Standard was physically broken up, Rockefeller maintained his one-quarter ownership in the business, only this time it was one-quarter of thirty-some new companies. J.P. Morgan was said to have remarked, "How the hell is any court going to compel a man to compete with himself?" The new set of companies continued to dominate the markets well into the 1930s." So more evidence of how the dissolution was handled. Stock exchange from the old Trust to the new individual companies seems like how it happened. --Jayron32 05:10, 26 October 2012 (UTC)[reply]
Read this book starting on page 106-107 largely confirms the last two sources. --Jayron32 05:18, 26 October 2012 (UTC)[reply]

Geographical information on ancient cities

I am looking for geographical information on ancient cities. Primarily urban layout maps of cities built around large stone temples or acropoli. I am wondering about any large stone structure that is centrally located in an urban area, such as greek cities. For example, I am to understand that scholars believe the Egyptian pyramids were built far away from cities rather than in them. Mayan pyramids, however, were built in the center of large cities settled on flat open ground. A large stone in the sunlight would get warmer than the textured urban area adjecent, and would therefore create a convection current which caused air to flow across the city even during times of zero or very low winds. I am researching the possibility that early humans may have been aware of the phenomenon and even planned it into their cities in order to improve sanitation. The greek acropolis might have had this effect, as would a pyramid or maybe even a castle. I am also looking for any information that might suggest this is a possibility, or not a possibility. Such an engineering design might become obsolete with the advent of flowing water, which would allow for a level of sanitation not possible before then, and therefore post roman era (in Europe) would cease to be important to residents of such an area. — Preceding unsigned comment added by 66.188.214.218 (talk) 04:04, 26 October 2012 (UTC)[reply]

I'm looking for information that would relate to the possibility that early human engineers were using large stone structures to create convection currents in the middle of urban areas on days with little to no wind. So, for example, a bunch of buildings have shade, indoors, fabric canopies, plants, etc. that would create a certain air temperature in sunlight, and the centrally located acropolis, pyramid, or other large temple would be exposed stone and therefore attain a higher temperature. This would cause the air in the urban area to flow, even at night as the heat stored in the stone was released, improving sanitation before flowing water became accessable via such means as the aqueduct. — Preceding unsigned comment added by 66.188.214.218 (talk) 04:14, 26 October 2012 (UTC)[reply]

That theory doesn't sound feasible to me. For one thing, you'd want the central rock/stone to be black, but it often seems to be white. Also, there are many other reasons for building up in the center, from defense in depth to impressing all the citizens living below. Also, air pollution wasn't much of a problem prior to industrialization and cities with huge, dense populations. StuRat (talk) 04:16, 26 October 2012 (UTC)[reply]

Ok, so the a white pyramid would work backwards, as well as provide access to upper air currents. And as to the smell of someone not taking a bath or shower, disposing of their waste in a chamber pot, owning several animals, and rarely doing laundry, I would say it might have caused health problems if the air wasn't being ventilated. So it could be possible that the smell was bad enough without the modern density. — Preceding unsigned comment added by 66.188.214.218 (talk) 04:39, 26 October 2012 (UTC)[reply]

(edit conflict) The theory fails Occam's razor in the sense that it proposes a complex and far-fetched explanation for events that have more commonplace and easy-to-understand explanations (sometimes this is known as the "Zebra solution": If you hear hooves, you shouldn't think "Zebra" instantly, you should think "Horse": look for the easier explanation, because it is usually right). In the case of cities built around hills, like Rome or the various Greek cities like Athens built around the Acropolis, the answer doesn't have anything to do with knowledge of convection currents. It has to do with knowledge of pointy sticks: Defending a static position is easier from the top of a hill than anywhere else. Large structures are often located within cities because large structures need lots of people to build them, and cities have lots of people in a central location: you don't have to move your labor force far away to build your temple or pyramid or whatever. Even the Giza Necropolis, though not at the center of a city, is not far from one. It was within easy distance of Memphis, the capital of the Old Kingdom and thus the administrative and commercial center of the Kingdom. Using a latitude/longitude calculator available online and the coordinates from our articles finds that Giza was 18 kilometers from Memphis, i.e. not downtown, but certainly within a reasonable distance to administer the work being done there. --Jayron32 04:42, 26 October 2012 (UTC)[reply]

One further question I consider in response. Suppose that there indeed was such a convection current (it would occur regardless of engineering, the heating patterns of a surface determine the effects of convection) and suppose it did indeed create a healthier more pleasant atmosphere for those in the central location, it might have caused the people who lived there to be more generally successful, as well as drive immigration by making an area more popular, making that city able to grow more than a competing city. — Preceding unsigned comment added by 66.188.214.218 (talk) 04:49, 26 October 2012 (UTC)[reply]

Well, again per the Zebra principle: Why search for an obscure, unlikely explanation for why people formed cities when there are a plethora of better reasons already availible. If you are generally interested in what actual historians have to actually say on the actual reasons why cities likely formed this page here [17] has a fairly easy to follow outline of the major factors in urban history and the development of cities, along with lots of good other sources to read. Wikipedia also has some good information at City#Origins. Does this help direct your thinking here? --Jayron32 05:47, 26 October 2012 (UTC)[reply]
There are plenty of ways to make your city more windy. Just putting it on a hill, like the Greek acropolis, would have made it much windier because no trees can block the wind. Building near a body of water helps, for the same reason the OP described, except that the heat sink is the entire ocean instead of a tiny piece of stone. See sea breeze and the corresponding land breeze. Narrow corridors like streets with buildings on both sides tend to channel the wind, which increases its speed. All of these effects dwarf any gain that could be had by placing a stone downtown. Needless to say, there are much more compelling reasons to build streets or locate your city on a hill and near an ocean. I'd be surprised if anybody who founded a city thought "oh, I'll build it on a hill to make it windier", instead of "I'll build it on a hill so the other dude doesn't kill me with his army".
Semi-random aside, because I've lately been obsessed with the Trojan War: the city of Troy was built on a hill, in between 2 rivers near their confluence, and beside a bay that extended from the Dardanelles (the bay has since dried up). The gods must have been with them, because that's just an amazing strategic location. Homer frequently calls the city "windy Troy". Gee, I wonder why... --140.180.252.244 (talk) 08:18, 26 October 2012 (UTC)[reply]
Probably because it was windy. Meanwhile, the story makes it clear that the enemy had a tough time penetrating Troy due to those geographic features. Throughout history, cities have been built on high ground, primarily for defensive reasons. This was the practice for thousands of years. It was compromised significantly once gunpowder and cannons were invented. ←Baseball Bugs What's up, Doc? carrots13:45, 26 October 2012 (UTC)[reply]
Not really. Well into the "gunpowder age", armies which maintained the high ground maintained an advantage in battle. Consider examples like Fortification of Dorchester Heights where a vastly undermanned American army was able to drive a much better manned, better armed, and better trained British army out of Boston merely by getting to the highest point in the area. Or the Battle of Bunker Hill a nominal "British" victory, but one which was clearly Pyrrhic: the "losing" Americans exacted a heavy toll on the advancing British forces marching up the hill before their retreat. The cases where high ground didn't provide an advantage are often due to incompetence in leadership, more than anything, i.e. Battle of the Little Bighorn, where Custer and his lieutenants had a combination of poor strategic decisions leading up to and during the battle, and where a tragic lack of coordination in the command structure led to the American forces being crushed despite holding the high ground. Cases of military incompetence aside, ground forces have always had an easier time defending a static position from a place of high ground than their assaulters, and likewise attacking downhill provides a significant advantage. In the here-and-now holding high ground doesn't necessarily have an advantage, but that's because the nature of warfare is such that it isn't based on armies in the open field engaged in pitched battle. Warfare today is about flushing small bands of enemies out of positions of hiding, or using such small bands to harass ones enemy from a concealed position. But even today, if I was trying to beat another large army of men with rifles with my own large army of men with rifles in open battle, I'd want to be the one on top of the hill and not the one at the bottom. --Jayron32 14:24, 26 October 2012 (UTC)[reply]
Just a sidenote—as I understand, it is quite unclear how the Battle of the Little Bighorn went. The traditional story of Custer and his men making a "last stand" on what is now called "Last Stand Hill" is certainly mythologized and probably wrong. It is likely that the main battle took place in the valley with various routed groups making "last stands" in various places, including hills but also gulches. Custer himself may well have been killed at Minneconjou Ford and his body taken to the hilltop later (perhaps by his own troops). In any case, I though I'd point out how this particular battle may not be the best example of defeat "despite holding the high ground". The historiography of the battle is interesting though, and a number of books have been written about it (the changing history of our understanding of the battle). Pfly (talk) 05:14, 27 October 2012 (UTC)[reply]
Agreed; consider the Battle of Monte Cassino in 1944. However, in earlier times, the need for an adequate water supply was a factor against hilltop fortifications. The other consideration is that a city needs commerce to survive, and so a sea or river frontage was usually a requirement. Alansplodge (talk) 14:54, 26 October 2012 (UTC)[reply]
Well, rivers aren't strictly required, but water supply is a key thing to having a defensible city. Ancient Jerusalem was cited where it was, both because it was on top of a hill and because of its access to reliable groundwater supplies. Jerusalem is an excellent example of a defensible city for those reasons: the Gihon Spring was able to supply Jerusalem with the waters it needed, and its position on the hill made it very defensible, despite being neither coastal nor on a river. Jerusalem's commercial importance came from its position along the Kidron Valley which was part of a series of natural "roads" through the area, and thus Jerusalem's position at a high point along the valley made it a key place to locate a fortified city to tap and control regional trade. --Jayron32 15:16, 26 October 2012 (UTC)[reply]
Ok, you've got me there. Alansplodge (talk) 19:02, 26 October 2012 (UTC)[reply]
Let me explicitly list some reasons why a hill is more defensible:
1) Increases the range of your weapons and decreases the range of enemy weapons. This is true of arrows and artillery, but less significant for bullets.
2) Your forces not engaged in fighting can hide on the far side of the hill during enemy fire, if the enemy is on one side only.
3) If your enemy charges up the hill, the sheer effort of scaling the hill will slow them down and cause fatigue. If steep enough, a fall can injure or kill the enemy. If even steeper, ladders may be required.
4) There's the possibility of using gravity weapons, like boulders rolled down the hill, or flaming logs.
5) It's difficult to use poison gas against such a target, as it will tend to either sink back down, rise up, or blow away.
6) If the hill has outcropping of rocks, those can be carved out to build bunkers. This is also possible in a valley, but rock outcroppings are more common on hills, as dirt tends to settle in low spots and cover rocks there. StuRat (talk) 19:27, 26 October 2012 (UTC)[reply]
We have an article on acropoleis in general, which is not very informative, and some small articles on the Acropolis of Rhodes and the Acrocorinth. The Acropolis of Athens article has much more information. Cities were built on an acropolis mostly for practical reasons of defense. It's more difficult to attack a city on a hill (although not impossible, the Persians sacked the Athenian acropolis for example). Rome and Byzantium were also founded on a single hill, originally. Adam Bishop (talk) 10:32, 26 October 2012 (UTC)[reply]

Recognizing that indeed defense is an important factor, it must be considered that for the success of a settlement there are other needs, such as adequate resources and ineed a healthy atmosphere. wars lasted on very short periods of time and for all of the rest of the time the success of each citizen was based on peacetime activities. pillaging can only feed a raiding army during a raid. As to the desireability of other locations they were indeed settled first. coastal towns and cities and mountain towns and cities all existed by them time people started settling flat open ground, but there was nowhere else that was economically feasible to go. a temple or pyramid is hardly a mere rock, and if people were aware of the conditions of hte natural acropolis and how they were disirable, they may have wanted to create those conditions again. there is information to be found on microscale convective weather, and it would certainly have been occurring on the athenian acropolis. It would not create a wind, per se, but it would create the kind of airflow we create with our household ventilation. — Preceding unsigned comment added by 66.188.214.218 (talk) 14:39, 26 October 2012 (UTC)[reply]

Until modern sanitation, cities were notoriously unhealthy but the causes were not understood. See miasma theory. Rmhermen (talk) 16:58, 26 October 2012 (UTC)[reply]
(edit conflict) "Healthy atmosphere" sounds like a basic requirement, but cities like London pretty much fly in the face of that assumption. London has long been one of the major world cities, and it is basically built in a giant toilet bowl (i.e. Great Stink). If "fresh air" were such a necessity, or if nasty air were such a hindrance to development, London would have never grown into its massive proportions in the first place. Now, public health concerns in general did lead to London working very hard to clean up its air and water over time, but that came after, and not before, the growth of London into one of the world's great urban centers. Your hypothesis sounds plausible if you ignore actual data and history. But when you really look at how and when and where and why cities develop, having a "nice breeze" was never a primary motivation, either through conscious choice or by some "invisible hand", for locating a city or for a city's growth. It simply doesn't appear, from the actual historic references, to have been a factor. --Jayron32 17:02, 26 October 2012 (UTC)[reply]


Also note that clean water is far more important than clean air. If everyone just craps into the same water they drink from, with no water or sewage treatment, water-borne diseases can cut life expectancy in half. By comparison, relatively few people die from stinky air. StuRat (talk) 19:16, 26 October 2012 (UTC)[reply]

London became a major power long after the age of stone megaliths. I can imagine a place where if people had fewer technologies they might have been very high skilled in areas such as masonry, and building megaliths would have been easier than we make it out to be. If there were a major city on the nile and the economy grew significantly there would be a humongous growth in the population. Assuming a fair level of social development, the people might have tried to do things to improve the peacetime environment of their homes, and if it was readily apparent that living near a large stone construction, such a centrally located temple (probably reserved for the very wealthy) then they might build more. London as a power came from people outside of london being successful. The residents of london were subject to legendary poverty that could not have existed before that time. The more ancient empires might have had to have more success within their city limits. Clean water is important, but clean air still matters, and enourmously. If everyone doesn't crap in the water, where do they crap? where do they put food waste? Even buried crap stinks. London had more baths. Ancient poor peoples might have bathed for celebrations or rights of passage only in some places. Some days have virtually no flowing air. Near or in deserts this can last for a week or longer. Whole sections of town were evacuated even as recently as the aforementioned london due to gases from decay of sewage. This might have been encountered frequently in order for humans to learn about waste disposal. It might be that at a low enough level of technology (how expensive can wheels be?) living next to a landform that keeps the air flowing isn't much different than having a clean water source.

also I should add that these guys are right about defense, but there is lots of time and people have lots of other problems, and they solve them. — Preceding unsigned comment added by 66.188.214.218 (talk) 03:12, 27 October 2012 (UTC)[reply]

Child slavery in nineteenth-century America.

My son wants to know how children his own age (4 years) laboured as American slaves. Did their work differ significantly from that of grown slaves, both in the field and in the house? If so, how?

(I note our article's reference to Gwyn Campbell, "Children and slavery in the new world: A review," Slavery & Abolition, Aug 2006, Vol. 27 Issue 2, pp 261–285, but I have not yet had opportunity to read this.)

Any advice of interest to a 4-year-old is especially welcome.82.31.133.165 (talk) 12:06, 26 October 2012 (UTC)[reply]

Obviously they did not do hard agricultural field labor that would have been far beyond their physical strength. Sometimes they might have done light gathering or cleaning tasks that would have been within their abilities and attention spans. According to the Autobiography of Frederick Douglass, until he was 7 or 8, "I was not old enough to work in the field, and there being little else than field work to do, I had a great deal of leisure time. The most I had to do was to drive up the cows at evening, keep the fowls out of the garden, keep the front yard clean, and run of errands for my old master's daughter, Mrs. Lucretia Auld." AnonMoos (talk) 12:42, 26 October 2012 (UTC)[reply]
This source notes "Even small children and the elderly were not exempt from these long work hours." though it does not age "small children", so I don't know exactly what age (4 or 9 could both be described as "small"). This page states that "Slave children were sent into the fields at about 12 years of age where they worked from sun up to sun down." Though it doesn't note if other work or shorter hours were expected of younger children. This one states that "The combination of hard physical labor, corporal punishment, a diet often lacking nutritional value, and poor living conditions contributed to a very high infant mortality rate—at least 20 percent of the slave children died before the age of five—and a much lower life expectancy than southern whites.", though it doesn't strictly state what children younger than 5 died of: whether being overworked, or simply malnutrition and preventable disease. While none of those sources is strictly scholarly or detailed, I did find this source which appears to be much more so. It states "Generally, in the U.S. South, children entered field work between the ages of eight and 12." I hope that gives you some leads to follow. --Jayron32 12:56, 26 October 2012 (UTC)[reply]
Another account of a slave childhood is in the first chapter of Up from Slavery, Booker T. Washington’s autobiography. He says when he was too small for field work he still worked, and mentions “cleaning the yards, carrying water to the men in the fields, or going to the mill to which I used to take the corn, once a week, to be ground. … required to go to the "big house" at meal-times to fan the flies from the table by means of a large set of paper fans operated by a pulley … ” There are lots of details that would interest a child – for example, the corn often fell off the horse and since he was too small to reload it, he had to wait for a chance helpful passerby. He often didn’t get home until after dark. His family never had a sit-down meal together. He longed to try cookies, which he saw white children eating. He went barefoot but had a pair of wooden shoes, which were very noisy. He owned one shirt, made of a painful, scratchy material, and no trousers.
FYI, there’s a very interesting list of autobiographies of slaves (with the texts), here.
Also, googling for children's picture books about slavery brings up tons of age-appropriate suggestions. Taknaran (talk) 14:42, 26 October 2012 (UTC)[reply]
For background, and also an interesting story, I would recommend Harriet Jacobs' Incidents in the Life of of a Slave Girl, a slave narrative written in the 1850s and published in Britain in 1862. The short answer is probably "it varied widely". The Key to Uncle Tom's Cabin discusses the tearing apart of families. It is probably worth remembering that virtually everyone writing at the time had an axe to grind, and (some) more modern scholarly works go to great pains to avoid inheriting these biases. Rich Farmbrough, 18:25, 27 October 2012 (UTC).[reply]

In northern regions, where and how did people bury their dead during winter?

In the middle ages, and the old days in general, how and where did people bury their dead in the colder regions during winter? Obviously the ground was frozen, and likely covered by deep snow. If someone died during winter, what did they do with the corpse? You can't dig through frozen earth. Of course, some places probably had catacombs and crypts of some kind, but in places without such burial sites, what did they do?

Krikkert7 (talk) 15:13, 26 October 2012 (UTC)[reply]

See our previous answer: Wikipedia:Reference desk/Archives/Humanities/2010 August 6#Winter burials. Rmhermen (talk) 16:19, 26 October 2012 (UTC)[reply]

Legally Protestant status of the Church of England

Is the Church of England still officially Protestant according to the laws of the realm? According to Universalis Ecclesiae, 10 George IV c. 10 (no clue what its short title is/was) was a law promulgated under George IV that referred to the Church as being Protestant, although of course I understand that it might have been repealed or superseded since then. Nyttend (talk) 18:03, 26 October 2012 (UTC)[reply]

What does "protestant" mean anyways? In the broadest sense, "Protestant" can mean literally any non-Catholic western Christian denomination founded during or after the Reformation. There's not really any unifying theological definition beyond that. I'm pretty sure that the meaning of "Protestant" in regards to the C of E merely means "Does not recognize the authority of the Pope" and literally nothing else. I've seen several reliable discourses over the classification of Anglicanism and Episcopalianism, and from a theological perspective, it isn't a settled matter by any means. There are serious arguments to be made in both directions: That it is Protestant because it did begin as a rejection of the Church of Rome, or that it isn't protestant because the theology more closely allies with Catholicism than it does with Lutheran or Calvinist theologies. Protestant#Anglicans and Episcopalians has a brief discussion of the problem of classifying the Anglican Community as "protestant". Now, what this means for the legal status of the Church of England specifically within the Laws of the United Kingdom is another matter, but I am fairly certain that the term Protestant in that context merely means "Not under papal authority" and little else. --Jayron32 18:13, 26 October 2012 (UTC)[reply]
More: Anglicanism#Terminology states " In British parliamentary legislation referring to the English Established Church, it is described as the "Protestant Episcopal Church", thereby distinguishing it from the counterpart established "Protestant Presbyterian Church" in Scotland. High Churchmen, who objected to the term "Protestant"," That is, the term may have been imposed by Parliament on the church, but may not have been unilaterally accepted as valid by those within the church at the time of the legislation. --Jayron32 18:17, 26 October 2012 (UTC)[reply]
Even more: This blog post and the missive it discusses both seem to be HIGHLY relevent to the nature of this question. --Jayron32 18:19, 26 October 2012 (UTC)[reply]
And yet more: here is an interesting perspective from a single Anglican church on the distinction and use of the terms here. --Jayron32 18:22, 26 October 2012 (UTC)[reply]
At the Coronation of a British monarch, the new sovereign is asked (among other things); "Will you to the utmost of your power maintain in the United Kingdom the Protestant Reformed Religion established by law?" The required answer is "All this I promise to do."[18] Alansplodge (talk) 18:57, 26 October 2012 (UTC)[reply]
Although it's about as likely as my being elected Pope, what would happen if the monarch-to-be refused to take this oath? Would that essentially be a statement of abdication? ←Baseball Bugs What's up, Doc? carrots23:55, 26 October 2012 (UTC)[reply]
No. A British monarch cannot unilaterally abdicate per se. But if they refuse to abide by any condition of their monarchy, I suppose the powers that be would be forced to take action, which might in extremis include passing a law deeming the monarch to have abdicated. Note that the throne is never vacant. The new monarch accedes the instant the previous one dies (or otherwise ceases to be monarch), simply by operation of the law. The Coronation, which might be 12-18 months down the track, does not make them the monarch, but it may well test whether they're suitable to remain monarch. Realistically, if the monarch had any issues about supporting Protestantism, I think they'd have raised them well before the Coronation, and probably well before they ever became monarch in the first place. A very unlikely scenario, as you say. But Pope Bugs I has a certain ring to it. Just remember we've always been the closest of friends, and I expect you to fast track my canonisation when the time comes.  :) -- Jack of Oz [Talk] 00:23, 27 October 2012 (UTC)[reply]
You bet! Once I'm the Pope, you'll be at the top of the list. Saint Jack of Oz has a nice ring to it also. :) Oh, I forgot one thing. You'd have to be dead. :( :( :( But not to worry. I'll issue a decree that "going on walkabout" qualifies. :) ←Baseball Bugs What's up, Doc? carrots03:32, 27 October 2012 (UTC)[reply]
Bless you, my son. I'll now add you to the list of papabili, and you can add me to the list of santabili. What? No article? No matter, this is Wikipedia. Be bold and create one. :) -- Jack of Oz [Talk] 03:51, 27 October 2012 (UTC)[reply]
Thanks for the answers; unfortunately I didn't have time to dig for resources earlier, beyond reading the article that I cited above. I emphasised the legal nature of the question largely because I'm well aware of the concept of via media and was surprised at the possibility of the law calling the Church Protestant. I was also thinking of the coronation oath, but I forgot the actual words and thought that it was something like "the true reformed religion", with "reformed" obviously meaning non-Catholic, not Reformed churches. Nyttend (talk) 00:29, 27 October 2012 (UTC)[reply]
From the horse's mouth: The Church of England is the Established Church of England and has been since 1689. That link makes no mention of it ceasing to be Protestant. From the Queen's point of view: the Church of Scotland is the Scottish counterpart to the Church of England. There are no Established Churches in Northern Ireland or Wales, or any of the Commonwealth realms according to that link (and I think it's pretty authoritative). --TammyMoet (talk) 09:13, 27 October 2012 (UTC)[reply]
Yes, the Anglican churches in Ireland and Wales have been disestablished (in 1871 and 1920 respectively). The movement opposing this was called antidisestablishmentarianism providing generations of schoolchildren with "the longest word in the dictionary". Alansplodge (talk) 09:39, 27 October 2012 (UTC)[reply]

Omzwervingen door de eilandenwereld van den Grooten-oceaan

Omzwervingen door de eilandenwereld van den Grooten-oceaan seems to be a German version of Promenades en Océanie: les Tubuaï et l'archipel de Cook. Is there an English translation of this?--KAVEBEAR (talk) 18:15, 26 October 2012 (UTC)[reply]

It's certainly not German, but Dutch. What makes you think these are the same books?
edit: "Omzwervingen door de eilandenwereld van den Grooten-oceaan" seems to originate in the Dutch 'travel literature magazine' De Aarde en haar volken. - Lindert (talk) 19:24, 26 October 2012 (UTC)[reply]
Oh sorry, not the entire book but this chapter same as Promenades en Océanie: les Tubuaï et l'archipel de Cook found in Le Tour du monde, Volumes 50. Does anybody know if there is an English translation?--KAVEBEAR (talk) 01:38, 27 October 2012 (UTC)[reply]
If I'm translating it correctly, the english title could be "Circumnavigating through the island-world of the great ocean." Definitely dutch. Plasmic Physics (talk) 11:16, 27 October 2012 (UTC)[reply]
That's quite a literal translation, but seeing that "Grooten-oceaan" is written with a capital letter, it should be translated 'Pacific Ocean'. I think "Omzwervingen" is closer to 'roamings' or 'undirected travels'. - Lindert (talk) 13:19, 27 October 2012 (UTC)[reply]

I searched Google Books for "Edouard Petit" and "translated by" and nothing came up; had similar results in my university library system. Everything by Petit seems to be in French. 184.147.123.169 (talk) 14:20, 27 October 2012 (UTC)[reply]

Did the Kaiser plan to invade Britain?

I've been working on an article about the British Volunteer Training Corps (World War I), a sort of early Home Guard. There seems to have been plenty of preparations for an expected invasion, but I wondered if there were actually any German plans. Apparently, the French had plans before 1908, but I haven't been able to uncover anything from the Germans, even a study that showed it was impossible. Can anyone help please? Alansplodge (talk) 18:48, 26 October 2012 (UTC)[reply]

I can't find anything either, despite several searches using German search terms. I think that British fears of a German invasion were overblown and not based on any real plans by Germany, whose attitude toward Britain was defensive. Plans for a hypothetical German invasion may have been part of a project on the part of Britain's military establishment to spread fear and loathing of Germany and to justify military expenditures, just as exaggerated estimates of Soviet military power were used to justify military expenditures to the public in the United States and other western allies during the Cold War. Marco polo (talk) 19:59, 26 October 2012 (UTC)[reply]
Well, many thanks for trying. I found this alternate history site which has a scenario where Britain is invaded in 1915. It starts with the premise "...plans for invasion of Great Britain, in a possible future European war, were first proposed by the German Navy in 1912...". I wondered if this was factual or imagined by the author. Alansplodge (talk) 10:20, 27 October 2012 (UTC)[reply]
Our article at invasion literature implies that public fears of such invasion were much greater than any actual threat, but there's no specific cite regarding Germany's actual plans one way or the other. Matt Deres (talk) 19:57, 28 October 2012 (UTC)[reply]
While not plans, we can infer a bit about the Generalstab's proximate intentions regarding Britain from the actions they assigned their secret agents there. Christopher Andrew's The Defence of the Realm discusses the activities of many, and they all seem to be concerned with the movements of Royal Navy and the goings-on in ports. This is unlike what Abteilung III b had their people in France and behind enemy lines in the Low Countries, where they were more active in the hinterland, reporting on movements of troops and materiel. The few German agents who weren't quickly British agents too don't seem to have spent any time doing the things you'd expect would be necessary when preparing for an invasion of a whole country. They're not scouting beaches, planning railway sabotage, or plotting with fifth columnists and dissidents. This doesn't entirely refute the suggestion that there was such a plan, as Germany would need to neutralise the Grand Fleet before setting an army over the channel in wobbly Rhine barges, but I think it does show that if such a plan existed at all, it was very low down in the Generalstab's priorities. -- Finlay McWalterTalk 11:35, 27 October 2012 (UTC)[reply]
I'm pretty sure that the Germans had no plans to invade Britain during World War I. Doing so would have been pretty much impossible given the Royal Navy's greater strength, and Germany's near total lack of amphibious warfare capabilities. The British actually developed plans to invade Germany by sea, but this was rejected as being a doomed venture (from memory, Churchill was the key figure in having these plans developed and advocating the scheme). Nick-D (talk) 03:54, 28 October 2012 (UTC)[reply]
Agreed. The Kaiser would have to be on quite a roll to be able to invade the UK. StuRat (talk) 04:01, 28 October 2012 (UTC) [reply]
Thanks, but I understand the probabilities; I was looking for a reference for an article. Alansplodge (talk) 09:07, 28 October 2012 (UTC)[reply]
Apologies, that reads as rather blunt, but you get the gist... Alansplodge (talk) 19:21, 28 October 2012 (UTC)[reply]

Gubernatorial primary elections (U.S.)

Hello, I have two questions about the primary elections for governors: At which time (month) do the gubernatorial primaries start and end? Are they beginning in January and finish in June like presidential primaries? I need to know that for writing a book about a story with a (fictive) governor. I'm esepecially interested in the state of California because the story of my book is playing in that state; and, last but not least, have those primary elections also been held in the 1960s? If not, how have candidates/nomiees for the general election been elected? --78.52.186.187 (talk) 19:53, 26 October 2012 (UTC)[reply]

The dates vary from one state to another. Also, typically primary elections for governor are held on the same date together with primaries for other state offices. The date of the state primary may be different from the date of the presidential primary in a given state. In California, gubernatorial elections are offset from presidential elections by two years. There have been gubernatorial primaries in California since the early 20th century. Since at least the 1960s, gubernatorial primaries in California have always been held in early June. See this source on the 1966 election and this one on the 1962 election. However, in states other than California, state primaries have been held in different months. Marco polo (talk) 20:34, 26 October 2012 (UTC)[reply]
Thanks. When they were held in June, that means nothing happend much before, like in spring? --78.51.166.21 (talk) 20:38, 26 October 2012 (UTC)[reply]
When you say "nothing happened", what are you referring to?... I would assume that the various contenders for their party's nomination would be attending rallies, giving speeches and holding fundraisers throughout the spring.
One thing to remember: Primary elections in the US are Party based... and if the Gubernatorial candidate for a given party is running unopposed (within that party), there is no need for that party to hold a Gubernatorial primary election. And if this is true for all political parties, then a State might not even hold a Gubernatorial primary election. Blueboar (talk) 14:28, 27 October 2012 (UTC)[reply]

Which day of the week for elections, and why?

Here in Australia elections have always been held on a Saturday. That made a lot of sense to me as I was growing up because it avoided a work day for most people, and avoided the Sunday when your'e supposed to be going to church and resting. It also made it easier to find the thousands of people needed to operate the polling booths

The US election is on a Tuesday. I have a vague impression in my (repressed through old age) memory that UK elections are held on a week day too. What's "normal" around the world? Does holding elections on a week day restrict access to a vote for some classes of people? Is that deliberate? How did the non-Saturday voting days get chosen? (e.g Why Tuesday in the USA?)HiLo48 (talk) 21:42, 26 October 2012 (UTC)[reply]

Many countries hold elections on Sundays (or the equivalent day-off of the week), in order to enable higher participation. For example in Sweden, general elections always take place on the third Sunday of September. Many countries have special voting days for people working in police or army (as they would be on duty during the polling day). In the recent West Bank local election, the polling station at the small Samaritan community was open for an hour longer than all others as the regular polling fell on the Sabbath.
Do note that the US stands out amongst Western countries in restricting its citizens from voting in general elections. Whereas some European countries have mandatory voting, the US employs the system of voter registration (unheard of in other developed countries). --Soman (talk) 21:58, 26 October 2012 (UTC)[reply]
That last comment seems to be quite unrelated to the topic of this thread.
See Election Day (politics), Election Day (United Kingdom) and Election Day (United States). -- Jack of Oz [Talk] 22:14, 26 October 2012 (UTC)[reply]
Irrelevant to the topic but relevant to Soman's last sentence
The following discussion has been closed. Please do not modify it.
The comment is not only unrelated, but seems to be wrong unless Soman either means a specific form of voter registration or doesn't consider a number of countries most of the world considers developed. Voter registration mentions non automatic (albeit mandatory) registration in Australia (which also has mandatory voting) and the UK (a rather odd system of the head of household being the one registering which may be changed). New Zealand also has mandatory but non automatic voter registration (but not voting). As our article mentions, it's not generally needed in countries with Resident registration although some (or at least one non developed) countries still require voter registration despite resident registration. As I mentioned, the US system likely has differences from voter registration in other countries but Soman didn't specify that the specific voter registration system in the US is unique (besides as our article attests, there isn't one system in the US since it depends on the states) simply that the system of voter registration is unique. Nil Einne (talk) 09:48, 27 October 2012 (UTC)[reply]
You yourself point out that the British, Oz and NZ voter registrations are very different from the US system. The US system still stands out, as it effectively disenfranchises large sections of population from voting (I know Guatemala has a similar system, effectively depriving tens of thousands of indigenous inhabitants from voting). --Soman (talk) 16:30, 27 October 2012 (UTC)[reply]
But why are we discussing this issue at all? The topic is about which day of the week the election is held. Nothing to do with voter registration, (dis)enfranchisement of voters, or any of the zillions of other matters related to elections. I'm hatting this tangent. -- Jack of Oz [Talk] 19:14, 27 October 2012 (UTC)[reply]
I apologise for continuing this tangent but I strongly dislike inaccurate claims being made on the the RD and Soman did make the claim without any sources or any explaination and now seems to have even conceded that their initial claim was at least poorly worded. If people don't want their claims to be challenged, they're free not to make them. Nil Einne (talk)
Actually you're mistaken. I never 'point out that the British, Oz and NZ voter registrations are very different from the US system'. I simply pointed out there obviously is going to be some differences. There are also differences between the British, Australian and NZ registration systems. How different the US systems are from the other systems, nor do I know it stands out, baring the fact it's at a state level. I have head a lot of controversy regarding US voting laws but I've never investigated it enough to know how bad it is in truth and in any case it seems to depend on the state, and a lot of the controversy does not concern registration but other stuff like voter ID laws and banning some perceived low level convicts. In any case if you are going to make the claim, you should at least provide the sources and preferably make an accurate claim in the first place and also do it in an on topic discussion rather then bringing it up off topic. As it stands I've provided the only sources (admitedly not for NZ but here [19] [20] [21]), it seems the one which does stand out is the UK which per our article technically allows the head of the household to stop someone from voting. Nil Einne (talk) 05:45, 28 October 2012 (UTC)[reply]
It occured to me I hadn't read our article properly so I checked it out and can't see any discussion related to the US registration system which makes it particularly distinct or bad compared to the rest in general. It mentions stuff like the disenfrachment of convicts and mentions how extreme it can be in some cases (anyone with a felony conviction barred for life) although as I discussed earlier this is distinct from the requirement for registration (you can bar convicts from voting with automatic registration or without requiring registration). Although I recall from some documentary that some states used private companies and ended up removing a lot of people they shouldn't have preventing them from voting which it's probably fair to say is part of the requirement for registration. It also seems to suggest in some cases it's better then NZ, Australia or UK since some states allows registration on election day whereas the best case for the other 3 appears to be NZ which allows it up to the day before (compared to according to our article, 3 weeks before for Australia and possibly slightly over 2 weeks for the UK [22]). P.S. North Dakota doesn't even require registration so it's even more unclear what's so unique about the US's registration requirement. Nil Einne (talk) 06:40, 28 October 2012 (UTC)[reply]
In the UK General elections are traditionally held on a Thursday. No-one appears to know why. - Cucumber Mike (talk) 22:23, 26 October 2012 (UTC)[reply]
Thanks Jack and Mike. Very helpful. Those first two articles Jack referenced tell us what day voting occurs too, but also not why. The US article is excellent in that it tells us why - "In 1845, the United States was largely an agrarian society. Farmers often needed a full day to travel by horse-drawn vehicles to the county seat to vote. Tuesday was established as election day because it did not interfere with the Biblical Sabbath or with market day, which was on Wednesday in many towns." As with many other laws established so long ago (in many countries, not just the US), it's obviously an irrelevant reason today. But will it ever change? HiLo48 (talk) 22:26, 26 October 2012 (UTC)[reply]
NPR did a story on this just the other day. See this. Zoonoses (talk) 00:30, 27 October 2012 (UTC)[reply]

October 27

UK House of Commons trivia

Two questions:

  1. When do newly-elected MPs take office? Basically as soon as a winner is officially named by elections officials, or is there some set period (e.g. "Ten days after the election") between election and officially taking office?
  2. What's the salary for the Steward of the Chiltern Hundreds? Presumably there's something, or it wouldn't be good for resignation purposes; is it something like £1 per year?

I've looked around for both of these answers, finding such documents as this, but I wasn't able to find anything through Google or at UK House of Commons, Kissing hands, the Chiltern steward article, or anywhere else that I looked. Have to admit that these questions grew out of reading the thread just above this one. Nyttend (talk) 00:24, 27 October 2012 (UTC)[reply]

On Q. 1: I assume this is covered in Erskine May: Parliamentary Practice but I'm having trouble accessing an online version. Having worked in this part of the Australian bureaucracy, I can assure you that members of our lower house commence their service and are paid from election day, no matter how long it might take for the result in their division to become clear or be officially declared. See Odgers' House of Representatives Practice at "Members' remuneration and entitlements":
  • A Member is paid salary and allowances from and including the day of the election, ...
See also further down the page at "Titles accorded to members":
  • A Member's status as a Member does not depend on the meeting of the Parliament, nor on the Member taking his or her seat or making the oath or affirmation. A Member is technically regarded as a Member from the day of election—that is, when he or she is, in the words of the Constitution, ‘chosen by the people’. A new Member is entitled to use the title MP once this status is officially confirmed by the declaration of the poll.
Australia has a Westminster system modelled after the UK's, so I can only assume it's the same there. -- Jack of Oz [Talk] 01:17, 27 October 2012 (UTC)[reply]
Right, I'm going to have a bash at the Chiltern Hundreds question. For those playing along at home, the post of Steward of the Chiltern Hundreds is a sinecure used to permit members of the House of Commons to leave their posts. By tradition, MPs are forbidden to resign. This dates from a 1624 resolution that states that MPs have been entrusted to represent their constituencies, and are not at liberty to resign this trust. At this time, serving in parliament was seen much more as a duty than a privilege, so it was more necessary to 'force' MPs to serve.
However, under the 1701 Act of Settlement, no person who holds an Office of profit under the crown may continue to be an MP. Thus, any MP appointed to a position which entitles them to receive money from the Crown will cease to be an MP. In modern times, MPs use this loophole to resign, and MPs wishing to resign are appointed to either post of Steward of the Chiltern Hundreds or Steward of the Manor of Northstead. Appointments usually alternate between the two.
In terms of the actual 'salary' one receives for these roles, things are somewhat unclear. According to the Encyclopædia Britannica of 1911, "Up to the 19th century there was a nominal salary of 20S. attached to the post [of Steward of the Chiltern Hundreds]", but "There are no traces of any profits having ever been derived from the office [of Steward and Bailiff of the Manor of Northstead]". There's no mention of when in the C19th the salary was abolished.
The Mirror of Parliament, Volume 1 by John Henry Barrow (1839) says: "...a salary of 20l a year is attached to it, though the money is never received by any person accepting the office." (p.276). 20l (italic small-case "L") would be £20 in modern notation. Alansplodge (talk) 09:46, 28 October 2012 (UTC)[reply]
Isn't that 20/-? That is, 20 followed by a solidus, meaning 20 shillings or £1 in new money. See £sd. - Cucumber Mike (talk) 11:05, 28 October 2012 (UTC)[reply]
No, in the 19th century and before, pounds were signified by an italic "L" after the amount. See Old Bailey Proceedings Online; 4th February 1839. "...he had clothes which came to 2l. 4s. 6d., and 3l. 0s. 6d. in cash". Also Calendar of Treasury Books, Volume 8 - 1685-1689 "Petitioner has sworn before Baron Atkins that what has been recovered of the premises is only worth 5l. 10s. 0d. per an." There is some inconclusive discussion at Talk:Pound sign. Alansplodge (talk) 19:58, 28 October 2012 (UTC)[reply]
A new Steward of the Chiltern Hundreds used to cost £14 15/- in an inauguration fee, which at one point came from the profits on the land. There was also stamp duty of £2 on the writ of appointment. The sale of land led to the estates becoming much less valuable and the Government paid the fee (see Gentleman's Magazine, January 1841 p. 43). There is a note in National Archives file E 197/1, the Crown book of appointments to the Stewardships from 1772 to 1847, which states "Fee of £16·15 on Appointment to the Stewardship of the Chiltern Hundreds discontinued according to Mr. Trevelyan's letter to me of the 22 Jan. 1840. R.B.A." The signature is presumably that of R.B. Adderley of the Exchequer Seal Office. Perhaps significantly, Trevelyan had only started work on the previous day. Sam Blacketer (talk) 11:53, 28 October 2012 (UTC)[reply]
According to the House of Commons Background Paper: Resignation from the House of Commons "Other such offices have been used for this purpose in the past, and some of them have carried duties and salaries: this is not the case today." This would suggest that the Stewards do not now receive any money following their appointments, nominal or otherwise.
I think the way this works is that by being appointed to one of these posts, MPs are entitled to receive the duties attached to the manors. The fact that, for at least a few hundred years, those duties have been £0.00 (or £0-0-0) is immaterial. It's the 'entitlement' to the profit that makes one inelegible to sit in the House. - Cucumber Mike (talk) 11:55, 27 October 2012 (UTC)[reply]
According to the Parliamentary fact sheet, "During the seventeenth century, a hundred years after any records of their actual administration cease, the office of Steward became divorced from any former actual duties, and ceased to enjoy any revenues from the area." So it's an "office for profit", but any profit has been stripped from it. It's the UK. It does not have to make sense ;-). --Stephan Schulz (talk) 12:58, 27 October 2012 (UTC)[reply]
On when a person elected becomes a Member of Parliament, elections are formally ordered by a Writ being sent from the Clerk of the Crown in Chancery to the Returning Officer in each Parliamentary constituency. The UK doesn't have any arrangement whereby absentee postal votes which arrive after polling stations close can be added to the count, so (unusually in a Western democracy) the count which takes place from 10 PM is the final result (save if it is challenged through an election petition). The official in charge of the election is called the 'Returning Officer' because the duty is that of making a Return to the Writ, which is to send back to the Clerk of the Crown the name and address of the person who has been duly elected. The Writ also formally tells the Returning Officer the date, time and place when the Parliament is summoned to meet. The Returns are all gathered together in a large white book and about ten minutes before the new Parliament meets, the book is formally handed over to the Clerk of the House of Commons. (See Votes and Proceedings for the record of this handover happening in the current Parliament.) Each person elected to Parliament then has to take the oath or affirm allegiance to the Crown in order to take their seat.
It is difficult to be precise about when a newly elected person has formally become a Member. It is certainly possible to argue that it is as soon as the Returning Officer has formally declared them elected. Against this it could be argued that no-one can be a member of a Parliament which has not yet been summoned, so it would be as soon as the Parliament had met. Members of Parliament are not paid unless they have taken the oath, but when they do take the oath their pay is backdated to the date their Return was received (see Briefing on Parliamentary Oath, p 8). However Members who have not taken the oath are still Members of Parliament and able to perform some functions. Sam Blacketer (talk) 13:23, 27 October 2012 (UTC)[reply]
I looked at two UK MPs chosen at random - Tony Blair and Patricia Hewitt. Blair was elected at the 1983 general election, held on 9 June. His own page tells me he was the Member for Sedgfield from, you guessed it, 9 June 1983. Hewitt was elected at at the 1997 election, held on 1 May. Her page says she was the Member for Leicester West from 1 May 1997. It seems clear that UK MPs are members from election day or by-election day as the case may be. If that were not the case, we'd have some explaining to do about Blair, Hewitt et no doubt al. -- Jack of Oz [Talk] 21:02, 27 October 2012 (UTC)[reply]
That seems to be the answer to a subtly different question - "When should Wikipedia count a Parliamentarian's term as beginning?" It is custom and practice here as elsewhere to refer to the polling day in the general election. In fact only a very few fast-counting constituencies are able to produce a result in the two hours between the close of the poll and midnight. I have, though, undone an edit which used the fact that a byelection count had concluded after midnight as a reason for saying the successful candidate's term began on the following day. (There's more disagreement about when the term ends for an MP who is defeated: is it on polling day, or the day Parliament was dissolved? Practice seems to differ.) Sam Blacketer (talk) 23:52, 27 October 2012 (UTC)[reply]
The background paper downloadable from here talks about the Father of the House
  • The Father of the House heads the ‘seniority list’ of Members, a list of Members in order of their length of unbroken service. If two or more Members enter the House at the same election, each with unbroken service, their seniority is determined by the date and/or time they took the oath.
That is, for the purposes of determining who the Father of the House is, it may be necessary to look at when two or more members took the oath. But the implication is that, for general purposes, the term of service of an MP commences on their election day. -- Jack of Oz [Talk] 03:04, 28 October 2012 (UTC)[reply]
The disqualification attaching to offices of profit under the Crown is triggered if there is a possibility of payment through the office; it is not necessary for any actual payment to have been made. In 1955 there were a series of problems with Members of Parliament who had posts which were unpaid but where there was a theoretical entitlement to expenses; they were held to be disqualifying. As a result of the confusion the House of Commons Disqualification Act 1957 was passed which lists all the affected posts in a Schedule, so that there is no doubt. Sam Blacketer (talk) 13:23, 27 October 2012 (UTC)[reply]

diffrenciation and cost leadership

can a firm or business achieve diffrenciation and mantian a low cos leadership at the sametime? — Preceding unsigned comment added by 61.175.228.68 (talk) 01:19, 27 October 2012 (UTC)[reply]

Have a look at Differentiation (economics), Cost leadership and especially Porter generic strategies; does that help? 184.147.123.169 (talk) 14:25, 27 October 2012 (UTC)[reply]
Generally, a differentiation strategy will employ advertising and other means, which will increase cost, over, say, generic alternatives. However, there may be exceptions. For example, once a company finds itself with a near monopoly, the economy of scale may allow them to advertise and still remain the lowest cost producer. StuRat (talk) 23:34, 27 October 2012 (UTC)[reply]

Hebrew english dictionary transliteration

Is there a website that is a Hebrew-english and English-Hebrew with English transliteration? I am trying to translate some English words into Hebrew like strong, might and union but I have hard time reading Hebrew that's why I need transliteration. — Preceding unsigned comment added by 70.31.22.30 (talk) 02:54, 27 October 2012 (UTC)[reply]

I google [hebrew transliteration online] and there seems to be a number of possibilities. ←Baseball Bugs What's up, Doc? carrots03:28, 27 October 2012 (UTC)[reply]
Have no idea about on-line, but in the dead-tree world, "Webster's New World Hebrew Dictionary" by Hayim Baltsan (ISBN 0-671-88991-5) is aimed at people like you... AnonMoos (talk) 05:59, 27 October 2012 (UTC)[reply]
WikiBooks has the beginnings of an elementary Hebrew course with transliteration here, but as of now it is woefully incomplete. The Foreign Services Institute has a very comprehensive course in Hebrew, with transliteration and accompanying recorded classes in mp3 format (around twenty hours of such material, by my tally) here. It's particularly useful in figuring out the particular Modern Israeli articulations of phonemes (especially vowels, which are by no means interchangeable with any of their English counterparts), something that is often neglected with the other gratis courses out there. Also, it's not exactly transliteration, but you might try Milon and Foundation Stone (downloadable), both of which include audio files of Hebrew pronunciation. And if it's good-ol' Biblical Hebrew you're working with, most editions of Strong's I've seen have transliteration in one form or another. Evanh2008 (talk|contribs) 13:20, 27 October 2012 (UTC)[reply]
Those may be good resources, but if the original questioner wants to look up Hebrew words based on their modern Israeli pronunciations (ignoring the Hebrew alphabet), then the Baltsan book seems to be one of the few tools that allows this... AnonMoos (talk) 10:06, 28 October 2012 (UTC)[reply]
Google Translate is simple and easy if you want an ok translation. Otherwise, as someone noted above, just googling "Hebrew dictionary online" or "Hebrew translation online" will get tons of results. If you want specific words, feel free to ask me on my talkpage.
  • Strong = חזק (pronounced as chazak)
  • Might = עָצְמָה (pronounced as otz-mah. Note: if you're asking it as in "Might I go to the bathroom," it'd be האם, pronounced as ha-eem)
  • Union = איחוד (pronounced as ee-chood)
Hope it helps! --Jethro B 00:37, 28 October 2012 (UTC)[reply]
To help a bit more, I'll indicate the stressed syllables in all caps and substitute letters I believe are closer to the Hebrew sounds: kha-ZAK, ots-MAH, ee-KHOOD - though in the _trade_ union context, I'd suggest ee-GOOD (rhymes with "see food"; Morfix online H>E dico gives "unification, merger, unity" for ee-KHOOD). -- Deborahjay (talk) 18:44, 28 October 2012 (UTC)[reply]

London geography - Primrose Hill

This question is related to some content from the Regent's Park page, but there is no content dispute, I am just curious about the facts. In fact, the reason I am asking here is to a large extent because it seems no one cares over at Talk:Regent's Park.

Primrose Hill is a hill in London. At some point content was introduced into the Regent's Park article to say:

"Primrose Hill is a Royal Park and belongs to the Sovereign along with all the other Royal Parks of the Crown Estate.

The supposition that Primrose Hill is owned and maintained by the Corporation of London is an error that has been the subject of successful Crown litigation in both in the High Court and Court of Appeal"

Now, I am wondering about the facts behind these statements because 1) Primrose Hill is not listed as a Royal Park in London, and 2) the second sentence seems a bit non sequitur - is there really a supposition that Primrose Hill is owned by the Corporation of London? And has the Crown really sued the City in the High Court and the Court of Appeal? --PalaceGuard008 (Talk) 15:01, 27 October 2012 (UTC)[reply]

I'll take the first bit: Whilst Primrose Hill isn't a separate Royal Park in and of itself, it is a part of Regent's Park. This is confirmed by Camden Borough Council: "Regent’s Park and Primrose Hill are operated by Royal Parks", by this Royal Parks proposal for improvements to the summit and by the map of Regent's Park. I haven't found any evidence so far about the Corporation of London being involved, but these things sometimes get into the Wikipedia article by someone adding something they think they know, someone else reverting it, and the compromise being that the information is left in, qualified by 'it is often thought that...'. I'll keep looking. - Cucumber Mike (talk) 15:36, 27 October 2012 (UTC)[reply]
(EC) On 1), see THE REGENT’S PARK & PRIMROSE HILL OPERATIONS PLAN: January 2009 "The Regent’s Park and Primrose Hill are distinct but contiguous public parks jointly managed by The Royal Parks". However, on the Royal Parks website, there is no mention of Primrose Hill on the list on their Parks page, however the accompanying map shows "The Regents Park and Primrose Hill" but when you click on the link, you only get details of Regent's Park. Alansplodge (talk) 15:42, 27 October 2012 (UTC)[reply]
Old and New London: Volume 5: CHAPTER XXII PRIMROSE HILL AND CHALK FARM (1878) says; "In 1827, the provost and fellows of Eton (who owned Primrose Hill at the time) began to see that their property would soon become valuable, and they obtained an Act of Parliament (7 Geo. IV., c. 25, private), enabling them to grant leases of lands in the parishes of Hampstead and Marylebone. Soon after the accession of Queen Victoria, endeavours were made to obtain Primrose Hill for the Crown, and a public act was passed (5 and 6 Vict., c. 78), for effecting an exchange between Her Majesty and the provost and college of Eton. By this act Eton College received certain property at Eton, and gave up all their rights in the Hill." Still searching for mention of litigation. Alansplodge (talk) 16:05, 27 October 2012 (UTC)[reply]
Regent's Park and Primrose Hill by Martin Sheppard (2010) says that Primrose Hill was acquired in 1841 because of a proposed scheme to turn it into a cemetery (p.97). No mention of any court case though, I suggest a "Citation needed" tag would be in order. Alansplodge (talk) 16:39, 27 October 2012 (UTC)[reply]
Since I still haven't found the details of the alleged court case, I'll just mention that one way in which the Corporation of the City of London is involved with Primrose Hill is over the subject of the Protected views: nothing is allowed to be built such that it would block the view of the dome of St Paul's Cathedral, or of the Palace of Westminster, from the top of Primrose Hill. Consideration is also given to the backdrop: there are restrictions on what may be built behind the buildings mentioned as well. This document has more details. - Cucumber Mike (talk) 19:58, 27 October 2012 (UTC)[reply]

U.S. election timings

Hey all. I'm looking for an updated version of http://news.bbc.co.uk/1/hi/world/americas/3760822.stm (for any timezone) but my google-fu just isn't up to strength. Does anyone know where I can find one? Or are the timings all going to be the same? Also, that page equivalates "polls close" and "result declared" -- how good an approximation is that likely to be? I'm trying to organise a social event around the elections, you see :) Thanks, - Jarry1250 [Deliberation needed] 15:09, 27 October 2012 (UTC)[reply]

I would assume that the timings are going to be very close to the 2004 values (here's a less-readily-useful but updated 2012 reference). As for "how good an approximation" -- no one can really say. Only a few swing states are really in play, so none of the west coast states in the late brackets are likely to matter (in terms of dramatic uncertainty). Nate Silver of fivethirtyeight currently estimates a 50/50 chance that Ohio will determine the election -- so on that basis, Ohio closing at 0030 UTC may be the highlight of the evening. If it's a landslide, that might be clear within 30 minutes (and good enough for pundits to declare X the all-but-inevitable winner). On the other hand, votes might be close enough to trigger a repeat of the 2000 election and take weeks to resolve, or any other timetable in between. — Lomn 18:44, 27 October 2012 (UTC)[reply]
(ec) [23] The results of many races will be able to be reliably predicted by news networks soon after the polls are closed (or even before the polls close, based on exit polling. In closer races, the information will take several hours as the different precincts report there numbers (maybe up to 12 hours, but most probably within 3-6 hours). However, most news networks will continue to give a blow-by-blow account as different results come in, so it could still be interesting (if you find that sort of thing interesting). If the race is close enough to trigger a recount, then the results could take a long time. If it's for president, as in the Florida election recount, they'll probably still try to get a president sworn in by January. If it's for a lower office, as in the United States Senate election in Minnesota, 2008, it could go significantly longer. Buddy431 (talk) 18:55, 27 October 2012 (UTC)[reply]
(ec) Yes, Buddy431's version of the link is substantially better. My advice, after the above caveats, is that 0000 - 0300 UTC is probably the sweet spot. If Romney has not made substantial progress in the Obama-leaning swing states of Virginia, Ohio, Wisconsin, and Colorado (all of which close by 0200), Obama probably wins, even with the remaining states yet to close (as those are the last of the large swing states). If Obama takes Florida (the largest Romney-leaning swing state) and Ohio, that's also probably good enough to call it his way. — Lomn 19:03, 27 October 2012 (UTC)[reply]
Thanks all. If I called it an evening at, say, 03:45 UTC, then, the general consensus is that I'd go to bed confident of the result? - Jarry1250 [Deliberation needed] 11:11, 28 October 2012 (UTC)[reply]
Not if it goes like it did in 2000. In fact, CBS projected Bush the winner in Florida, and then had to backtrack as the situation became clearer (or muddier). It was a good stretch of time before they got the Florida thing figured out. And it could happen again. ←Baseball Bugs What's up, Doc? carrots16:59, 28 October 2012 (UTC)[reply]
Yes, barring a goes-to-recount result in a critical state in an otherwise-close election, I think the answer will be in hand by 0345. — Lomn 22:33, 28 October 2012 (UTC)[reply]

Freemasonry and homosexuality

My question is whether or not Freemasons accept homosexuals as member of their Lodges. Thank you. Iowafromiowa (talk) 19:02, 27 October 2012 (UTC)[reply]

A google search for "freemason homosexual membership" is instructive, provided you filter past the conspiracy-theorist results. The forums associated with AskAFreemason suggest "it depends on the lodge". This story from Lexington, KY, notes a vote to dismiss a proposal that would have banned gay members for that state. The presence of a vote at that level suggests to me that the initial link is on the right track: it depends on the lodge. — Lomn 19:08, 27 October 2012 (UTC)[reply]
Freemasonry, as an institution, accepts homosexuals. There is no Grand Lodge that has a ban on homosexuals being initiated. However, election to the fraternity takes place at the individual lodge level... so: "It depends on the individual lodge" is the correct answer. I personally know several several Freemasons who are openly homosexual. They joined lodges that either did not care about a candidate's sexual orientation, or actively sought out gay men. However, I also personally know several Freemasons who are openly homophobic. They tend to belong to lodges that consist of other homophobes... lodges that would reject an openly gay men (or even a man who the members suspect is gay).
It also can depend on what area of your state you are in. Freemasonry is a cross section of society in general. In a town or city where broader society is accepting of homosexuality, the local Masonic lodge will tend to reflect that societal acceptance. In a town or city where broader society is not accepting, the local Masonic lodge will reflect that lack of acceptance. Blueboar (talk) 20:46, 27 October 2012 (UTC)[reply]

Dividends and stock price

When people know upfront that some company is paying dividends, and I suppose it's almost always announced, how does that influence the price? I thought at first that the price would increase shortly afterbefore the payment day, and fall after that, but it seems that it follows a different logic. How does it work? Comploose (talk) 21:06, 27 October 2012 (UTC)[reply]

I'm not sure why you would think that. To me it seems more logical that the price should be highest shortly before the dividend payout, because there is going to be a very quick cash return. In most cases, though, the dividend is not a large enough fraction of the stock price to have a major short-term effect one way or the other. Looie496 (talk) 21:39, 27 October 2012 (UTC)[reply]
Sorry, I meant before. BTW, some dividends are higher than 5%, isn't that much? Comploose (talk) 22:49, 27 October 2012 (UTC)[reply]
Yes, but most dividends are paid quarterly, and the difference between getting 2% tomorrow and 2% three months from now is not all that much. Looie496 (talk) 03:40, 28 October 2012 (UTC)[reply]
The difference between getting the 2% tomorrow or in three months would be huge, if you could re-use your money for obtaining other 2% the day after tomorrow. But, things don't work like that. The expectation of getting 2% dividends for your stock is balanced by any other existent negative expectation. And these negative expectations exist if a company is paying dividends, since that's only due to the fact that there are good reasons to expect a negative outcome. That's way dividend paying stock is not getting up before they pay and down after. OsmanRF34 (talk) 17:52, 28 October 2012 (UTC)[reply]

Write-in Candidates

One of the earlier threads about the upcoming election in the US made me think about write-in candidates. Being from a country where such a thing would never be allowed I'm just curious as to how that works. In particular I'd like to know how the people who count the votes determine who was meant by the people who wrote in their choice. It probably has no relevance in the real world but imagine this scenario: John Doe was never a candidate for any office but thousands of people write him in on the ballot (by the way, how do you write a name in on those ballots that are actually punchcards?). So John Doe gets the majority of the votes and thus becomes mayor-elect (or whatever office we're talking about). The John Doe everybody had in mind now says he doesn't really want to be mayor and declines the office. Since there is no shortage of John Does in the City, some other John Doe goes up there and says: "My name is John Doe and John Doe was elected mayor and I'd be happy to assume that office." How do you legally stop him becoming mayor?--Zoppp (talk) 22:23, 27 October 2012 (UTC)[reply]

See the second paragraph of our article on Write-in candidates. I've copied it below.-gadfium 23:06, 27 October 2012 (UTC)[reply]
"Some jurisdictions require write-in candidates be registered as official candidates before the election. This is standard in elections with a large pool of potential candidates, as there may be multiple candidates with the same name that could be written in."
But not all jurisdictions require that. As I noted in the other thread, Pennsylvania has no such requirement. And while the scenario that Zoppp described is unlikely in a city, in a small community it does happen. In my town a few years ago, a man was elected to borough council by write-in vote. No one was running for the office, but there were several dozen write-in votes. Each person written in had one vote, except for one man who had 3 votes. No one had ever even campaigned for the seat. But since he had the most votes, he was on council.    → Michael J    03:15, 28 October 2012 (UTC)[reply]

October 28

Did some of the world war 2, Korean War and Vietnam soldiers lie about their age and are any of soldiers who lied are alive today?

I'm wondering if some the ww2 , Korean war soldiers and Vietnam soldiers lied about their age and they were 17 or 16 they lied and they lied said they were 18. I could be wrong about this but back then they didn't have photo ID like today so it was easy to lie about their age. A birth certificate was probably needed, but if the person could pass for the required age, they could get in. I think also the country was eager to get any volunteers they could to protect and fight for their country. They did this in ww1 but what about ww2, Korean War and the Vietnam war and are any those soldiers who lied are alive today? 00:08, 28 October 2012 (UTC)

The Veterans of Underage Military Service says it has 29 active members who served in the US military or merchant marine at age 13, plus others older. Clarityfiend (talk) 00:47, 28 October 2012 (UTC)[reply]
This February 2012 article gives Walter Holy's account of his enlistment at the age of 15 and service with the paratroopers. Clarityfiend (talk) 01:00, 28 October 2012 (UTC)[reply]
You'll need a pretty blind recruiting sergeant to let a 13 years old pass as 18. OsmanRF34 (talk) 01:18, 28 October 2012 (UTC)[reply]
Well, yes, but then even today minimum enlistment age in the US is 17 (with parental consent), not 18. And boy seamen were allowed into the Royal Navy at age 14, though I don't know the equivalent in the US. --Stephan Schulz (talk) 01:31, 28 October 2012 (UTC)[reply]
There are some pretty big 13 year olds.[24] Clarityfiend (talk) 05:09, 28 October 2012 (UTC)[reply]
And some young-looking 18 year-olds. Alansplodge (talk) 09:02, 28 October 2012 (UTC)[reply]

Maybe he's lying today to you and he's not a vet of the Korean war OsmanRF34 (talk) 18:00, 28 October 2012 (UTC)[reply]

I wonder about how long ago the OP thinks the Vietnam War was! The youngest American veterans (18 in early 1975) are only 55 years old now. I doubt there are many reliable sources about the youngest Vietnamese veterans. Roger (talk) 18:18, 28 October 2012 (UTC)[reply]
The Korean War is not the Vietnam War. --140.180.252.244 (talk) 19:57, 28 October 2012 (UTC)[reply]
See Dan Bullock "...the youngest American serviceman killed in action during the Vietnam War." He joined the Marine Corps at 14 and was killed at An Hoa Combat Base aged 15. Alansplodge (talk) 19:34, 28 October 2012 (UTC)[reply]
See also My Father's Story by Crissandra L. Turner, which says of Edward Turner Jr. "In 1953 he sneaked off and joined the army Airborne using a cousin’s name at the age of 14, making him the youngest Korean War Veteran from the State of Missouri." Alansplodge (talk) 20:12, 28 October 2012 (UTC)[reply]
The movie Too Young the Hero is the true story of a 12-year-old who joined the Navy during World War II. 69.62.243.48 (talk) 00:26, 29 October 2012 (UTC)[reply]

how much cropland is lost to circle farming?

how much cropland is lost to circle farming? Naively, we might assume that 2r*2r is the square equivalent versus pi * rsquared is the circle equivalent. But that would mean 4 rsquared versus 3.141 r squared - i.e. "almost a third" more area is actually available (1.27x more) if they farmed squares instad of farming circles. Is this right though? Because you don't need clearance of a full square around the circle, you can pack the next circle in slightly more closely. I'm asking about this.

http://www.google.com/search?q=circle+farming&tbm=isch

how much cropland is lost to circle famring instad of square farming? --89.132.116.35 (talk) 13:54, 28 October 2012 (UTC)[reply]

You'll want to read the article on Circle packing. I'm not entirely clear on the maths, but it seems that, if the circles are identical, by using hexagonal packing you can cover a maximum of about 0.9069 of the area - that is, a little less than a tenth of the area available is unplanted. - Cucumber Mike (talk) 14:34, 28 October 2012 (UTC)[reply]
Circle farming is generally associated with center pivot irrigation (CPI). Our article mentions that CPI uses less water and is less labour intensive / requires less maintenance than alternatives for irrigating rectangular areas (RA). As such, the increased cost of irrigating an RA must be juxtaposed with the 10% (hexagonal) / 30% (square) inceased yield of this RA.
BTW, some further research indicates that the "left-over corners" of circle farms are are, ia, used for storing liquid fertilisers in (quite large) pools which are fed into the CPI system. Also bear in mind that circle farming is used in areas which were useless to agriculture before irrigation became feasible. This was never fertile cropland. --Cookatoo.ergo.ZooM (talk) 20:52, 28 October 2012 (UTC)[reply]

Are there any black people in Greenland living there?

Are there any black people in Greenland living there? Neptunekh94 (talk) 19:03, 28 October 2012 (UTC)[reply]

I would assume that there are black people stationed at Thule Air Base. Although that may not be what you are talking about (as the service men living at the base there are not permanent residents). Blueboar (talk) 20:54, 28 October 2012 (UTC)[reply]
88% of Greenland's population is Inuit, and the Inuit are kind of black. Do they count? --140.180.252.244 (talk) 22:47, 28 October 2012 (UTC)[reply]
There are descendents of Matthew Henson, who are also Inuit.[25] Clarityfiend (talk) 22:52, 28 October 2012 (UTC)[reply]

Tete-Michel Kpomassie, 1981, An African in Greenland. — kwami (talk) 23:06, 28 October 2012 (UTC)[reply]

US capital gains tax

I'm curious about the definition of "capital gains tax" in the U.S. How does the IRS treat the income made by partners in a professional partnership like a law firm? Does it treat it like dividend income (and hence subject to a lower rate) or is it considered ordinary income like a salary? Thanks. — Preceding unsigned comment added by 94.99.100.234 (talk) 20:10, 28 October 2012 (UTC)[reply]

In a partnership, income generally retains its character, and the distribution to partners (or failure to do so) is a non-event for tax purposes. This has led to controversy in the case of carried interest, when distributions to a hedge fund manager are treated as capital gains, even though the manager is receiving the distributions because of its provision of professional services to the partnership. John M Baker (talk) 21:02, 28 October 2012 (UTC)[reply]